Сколько вариантов значений может принимать 1 байт: Байт вмещает 256 символов? — Хабр Q&A

Содержание

256 бит равно байт — Все о Windows 10

На чтение 5 мин. Просмотров 143 Опубликовано

1 Байт = 8 бит, т.е. это строка 1,2,3,4,5,6,7,8 почему тогда 2 возводят в 8-ю степень и получают 256 ? И что с этим делать? Т.е. каким образом и для чего записываются данные?

P.S. Извиняюсь за столь детский вопрос, но я никак не могу понять зачем это действие нужно?

Ваши рассуждения не совсем точны.

В байте действительно 8 бит, но нумеруются они от 0 до 7, причем нумерация ведется справа налево по возрастанию. Каждый такой номер является «весом» для бита равного 1, а вес этот измеряется соответствующей степенью числа 2 (биты это двоичная система счисления, где возможны только варианты 0 или 1).

Таким образом, максимально возможное число, которое можно отразить одним байтом в двоичной системе счисления, составляет 8 подряд идущих единичных битов, каждый со своим весом: 1*2^0 + 1*2^1 + 1*2^2 + 1*2^3 + 1*2^4 + 1*2^5 + 1*2^6 + 1*2^7

или 1+2+4+8+16+32+64+128 = 255. Но еще осталось значение 0, соответствующее случаю, когда все 8 бит байта нулевые. Следовательно, максимум кодов какие можно получить с помощью одного байта составляет 255+1 = 256 кодов (символов).

1 байт не может вместить 256 символов, одним байтом можно закодировать любой один из 256-ти символов, потому что именно столько уникальных комбинаций может принять последовательность из 8-ми двоичных бит. Один двоичный бит — это наименьшая единица количества информации, он может принять лишь два значения 0 и 1. Последовательность двух бит может принят уже 4 значения: 00, 01, 10 и 11, а из трех бит 8 значений, 000, 001 и так далее. Добавление каждого бита увеличивает количество возможных значений, которая может принять битовая последовательность в два раза, соответственно последовательность из 8 бит сможет принять 256 различных значений. Поэтому и используют степени двойки, так как 2 в степени N равно тому, сколько значений может принять последовательность из N двоичных бит.

Далее, существуют кодовые таблицы (ASCII, Win-1251 и т.п.), в которых каждым символам, таким как: большие и маленькие буквы английского и национального алфавитов, цифры, знаки препинания и спецсимволы, соответствует определенное значение байта, например для символа Q — это 81, соответственно 01010001. И всего в таблице и есть 256 символов, но при этом одним байтом можно «написать» один символ, а что-бы его потом прочитать, необходимо знать какая кодовая таблица, так называемая «кодировка», использовалась.

Но если это Ваша очепятка, и речь идет о Байтах, тогда извольте:

а) Если байт считать октетом, то 1 Байт равен 8 битам ( 1 Б = 8 б )

б) 1 КилоБайт равен 1024 Байтам ( 1 КБ=1024 Б )

в) 1 МегаБайт равен 1024 КилоБайтам ( 1 МБ = 1024 КБ)

г) 256 МегаБайт равны 2 147 483 648 битам ( 256 МБ * 1024 КБ * 1024 Б * 8 б = 268435456 Б * 8 б = 2147483648 б)

Ответ: В 256 МегаБайт содержится 2147483648 бит.

Что такое 256-битное шифрование? Это то, что каждый центр сертификации и его реселлеры говорят при рекламе своих SSL-сертификатов. А почему бы и нет? В конце концов, шифрование является самым важным в SSL-сертификатах. Но многие пользователи, в том числе некоторые из клиентов нашей компании, не знают о действительной пользе, которую дает 256-битное шифрование.

Итак, давайте разберем его для Вас. Давайте разгадаем технологию, которая имеет 256-битное шифрование и поймем, как она защищает всех в Интернете.

Во-первых, давайте разберем SSL-шифрование

Принято считать, что SSL-шифрование выполняется с помощью пары ключей, известной как Public / Private Key Pair. Также считается, что открытый ключ шифрует данные, а Private Key расшифровывает его. Технически это называется «Асимметричное шифрование».

Однако, это не совсем так.

Фактическое шифрование данных не выполняется с помощью этого асимметричного метода; это делается с помощью Symmetric Encryption. Теперь следующий вопрос в вашей голове должен быть следующим: «Тогда почему используется пара Public / Private Key?» Да, он используется, но только для аутентификации. Когда клиент и сервер сталкиваются друг с другом, им необходимо проверить личность друг друга. Именно здесь играет роль асимметричного шифрования. Эта проверка выполняется с помощью пары Public / Private Key. После того, как аутентификация будет выполнена, и будет выполнено TLS handshake, начнется фактическое шифрование, и будет оно выполняться через Symmetric Encryption.

Что такое 256-битное шифрование?

Сначала термин «256-бит» звучит, как язык для «ботаников». Ну, это не так. 256-битное шифрование относится к длине ключа симметричной технологии шифрования. Это означает, что ключ сделан из 256 двоичных файлов (нулей и единиц), и имеется 2 256 возможных комбинаций.

Все еще не понятно? Хорошо, давайте разберем это с помощью простого примера. Допустим, есть 2-битный ключ. Следовательно, он будет иметь 2 2 (4) значения — 00, 01, 10 и 11. Таким образом, 256-битный ключ может иметь 2 256 возможных комбинаций. Ну что, так понятнее?)

Насколько безопасно 256-битное шифрование?

«Насколько безопасно 256-битное шифрование?» «Достаточно ли 256-битного шифрования?» Это два из наиболее часто задаваемых вопросов, когда дело доходит до уровня шифрования. Проясним раз и навсегда: этого более чем достаточно. Наша уверенность вполне обоснована и причина всему называется «Математика».

Лучший способ взломать ключ шифрования, и это «грубая форсировка», пробная версия и ошибка в простых терминах. Таким образом, если длина ключа составляет 256 бит, было бы доступно по 2 256 возможных комбинаций, и хакер должен попробовать большинство из 2 255 возможных комбинаций, прежде чем прийти к заключению (как правило, для получения правильной комбинации требуется около 50% ключей).

На бумаге 2 256 может показаться обычным числом, но не смейте недооценивать математику.

256-бит будет иметь такой вид:

115,792,089,237,316,195,423,570,985,008,687,907,853,269,984,665,640,564,039, 457,540,007,913,129,639,936 (78 цифр) возможных комбинаций.

Никакой суперкомпьютер на нашей с вами любимой планете не сможет взломать его.

Даже если вы будете использовать Tianhe-2 (MilkyWay-2), самый быстрый суперкомпьютер в мире, понадобится миллионы лет, чтобы взломать 256-битное шифрование.

Ну что, теперь вы чувствуете себя безопаснее?)

1 байт 8 бит. Сколько бит в байте? Что такое бит и байт

Чтобы досканально разобраться что такое Биты, что такое Байты и зачем всё это нужно, давайте сначала стоит немного остановимся на понятии «Информация», так как именно на ней построена работа вычислительной техники и сетей передачи данных, в том числе и нашего любимого Интернета.
Для человека, Информация — это некие знания или сведения, которыми обмениваются люди в процессе общения. Сначала знаниями обменивались устно, передавая друг другу, затем появилась письменность и информацию стали передавать уже с помощью рукописей, а затем уже и книг. Для вычислительных систем Информация — это данные которые собираются, обрабатываются, сохраняются и передаются дальше между звеньями системы, либо между разными компьютерными системами. Но если раньше информация помещалась в книги и её объём можно было хоть как-то наглядно оценить, например в библиотеке, то в условиях цифровых технологий она стала вирутальной и её нельзя измерить с помощью обычной и привычной метрической системы, к которой мы привыкли. Поэтому были введены единицы измерения информации — Биты и Байты.

Бит информации

В компьютере информация хранится на специальных носителях. Вот самые основные и знакомые большинству из нас:

Жесткий диск (HDD, SSD) — оптический диск (CD, DVD) — съёмные USB-диски (флешки, USB-HDD) — карты памяти (SD, microSD и т.п.)

Ваш персональный компьютер или ноутбук получает информацию, в основном в виде файлов с различным объёмом данных. Каждый из этих файлов любой носитель данных на аппаратном уровне получает, обрабатывает, хранит и передаёт в виде последовательности сигналов. Есть сигнал — единица, нет сигнала — ноль. Таким образом вся храняшаяся на жестком диске информация — документы, музыка, фильмы, игры — предствалена в виде нулей: 0 и единиц: 1. Эта система исчисления называется двоичной (используется всего два числа).

Вот одна единица информации (без разницы 0 это или 1) и называеся бит . Само слово bit пришло к нам как аббревиатура от bi nary digit — двоичное число. Что примечательно, в английском языке есть слово bit — немного, кусочек. Таким образом, бит — это самая наименьшая единица объёма информации.

Сколько битов в Байте

Как Вы уже поняли выше, сам по себе, бит — это самая маленькая единица в системе измерения информации. Оттого и пользоваться ею совсем неудобно. В итоге, в 1956 году Владимир Бухгольц ввёл ещё одну единицу измерения — Байт , как пучок из 8 бит. Вот наглядный пример байта в двоичной системе:

00000001 10000000 11111111

Таким образом, вот эти 8 бит и есть Байт. Он представляет собой комбинацию из 8 цифр, каждая из которых может быть либо единицей, либо нулем. Всего получается 256 комбинаций. Вот как то так.

Килобайт, Мегабайт, Гигабайт

Со временем, объёмы информации росли, причём в последние годы в геометрической прогрессии. Поэтому, решено было использовать приставки метрической системы СИ: Кило, Мега, Гига, Тера и т.п.
Приставка «кило» означает 1000, приставка «мега» подразумевает миллион, «гига» — миллиард и т.д. При этом нельзя проводить аналогии между обычным килобитом и килобайтом. Дело в том, что килобайт — это отнюдь не тысяча байт, а 2 в 10-й степени, то есть 1024 байт.

Соответственно, мегабайт — это 1024 килобайт или 1048576 байт.
Гигабайт получается равен 1024 мегабайт или 1048576 килобайт или 1073741824 байт.

Для простоты можно использовать такую таблицу:

Для примера хочу привести вот такие цифры:
Стандартный лист А4 с печатным текстом занимает в средем около 100 килобайт
Обычная фотография на простой цифровой фотоаппарат — 5-8 мегабайт

Фотографии, сделанные на профессиональный фотоаппарат — 12-18 мегабайт
Музыкальный трек формата mp3 среднего качества на 5 минут — около 10 мегабайт.
Обычный фильм на 90 минут, сжатый в обычном качестве — 1,5-2 гигабайта
Тот же фильм в HD-качестве — от 20 до 40 гигабайт.

P.S.:
Теперь отвечу на вопросы, которые мне наиболее часто задают новички.
1. Сколько Килобит в Мегабите? Ответ — 1000 килобит (по системе СИ)
2.6 (или миллион по-другому). То есть относительно байта 1 Мб = 1000000 б. Это самая распространенная на сегодняшний момент величина, которая характеризует размер множества файлов. Однако стоит сказать, что в скором времени возможен относительный переход на другие объемы занимаемой памяти, что повлечет за собой переход на другие приставки как основные. Методом логики и небольших естественных знаний мы получили, сколько байт в мегабайте. Их около миллиона.

Пришла пора обратиться к главному вопросу, к ответу на который мы плавно подошли. Сначала минутка математики:

1 кб = 1024 б;

1 мб = 1000000 б;

1 мб = 1000 кб.

Теперь уже при помощи математического метода был дан ответ на вопрос о том, сколько в мегабайте килобайт. Как видите, здесь нет ничего особенного. Обыкновенные расчеты помогут вам в трудной ситуации. Чтобы не быть голословными, рассмотрим на примере обычную школьную задачу.

Предположим, что ваш диск имеет свободное место в 200 Мб. А вам необходимо разместить на нем текстовые файлы размеров в 700 Кб. Количество их должно стремиться к бесконечности (такое вот условие), но оно ограничено размером. Вопрос прост: сколько у вас получится сохранить документов?

Решение выглядит следующим образом. Для начала вы вспоминаете, сколько в мегабайте килобайт. На этом этапе в голову должна прийти правильная мысль, что 1 Мб = 1024 Кб. Дальше вы понимаете, что в вашем распоряжении 200*1024 = 204800 Кб. Это число делится на размер файлов. То есть n = = 292. Квадратные скобочки обозначают целую часть числа, так как файл можно вставлять только полностью, не изменяя его размер. Ответом является число n. Этот простой пример лишь показывает то, как на практике можно применить знания о том, сколько в мегабайте килобайт.

Таким образом, вы получили ответ на поставленный ранее вопрос. Он иллюстрирует то, что в компьютерах нет ничего сложного. Все то, что с ними связано, можно посчитать, воспользовавшись не самыми глубокими познаниями.

Все фотографии, текстовые документы и программы хранятся в компьютерной памяти в виде битов и байтов. Что представляют собой эти мельчайшие единицы информации и сколько бит в байте?

Хранение данных в памяти

Компьютерная память представляет собой огромный набор ячеек, наполненных нулями и единицами. Ячейка — это минимальный объем данных, к которому может обращаться считывающее устройство. Физически она представляет собой триггер (в современных компьютерах). Триггер настолько мал, что его сложно рассмотреть даже под микроскопом. У каждой ячейки есть уникальный адрес, по которому ее находит та или иная программа.

Под ячейкой в большинстве случаев понимают один байт. Но, в зависимости от разрядности архитектуры, она может объединять в себе 2, 4 или 8 байт. Байт воспринимается электронными устройствами как единое целое, но на самом деле он состоит из еще меньших ячеек — битов. В 1 байте можно закодировать какой-нибудь символ, например, букву или цифру, в то время как 1 бита для этого недостаточно.

Контроллеры редко оперируют отдельными битами, хотя технически это возможно. Вместо этого идет обращение к целым байтам или даже группам байтов.

Что такое бит?

Часто под битом понимают единицу измерения информации. Такое определение нельзя назвать точным, потому что само понятие информации достаточно размыто. Если говорить более корректно, то бит — это буква компьютерного алфавита. Слово «бит» происходит от английского выражения «binary digit», что дословно означает «двоичная цифра».

Алфавит компьютеров прост и состоит всего из двух символов: 1 и 0 (наличие или отсутствие сигнала, истина или ложь). Этого набора вполне достаточно, чтобы логически описать все, что угодно. Третье состояние, под которым понимают молчание компьютера (прекращение передачи сигналов), является мифом.

Сама по себе буква не несет в себе никакой ценности с точки зрения информации: глядя на единицу или ноль, невозможно понять даже то, к какого рода данным это значение относится. И фото, и тексты, и программы в конечном счете состоят из единиц и нулей. Поэтому бит неудобен в качестве самостоятельной единицы. Следовательно, биты необходимо объединять для того, чтобы кодировать с их помощью полезную информацию.

Что такое байт?

Если бит — это буква, то байт представляет собой подобие слова. Один байт может содержать текстовый символ, целое число, часть большого числа, два небольших числа и т. д. Таким образом, в байте уже содержится осмысленная информация, хоть и в небольшом объеме.

Начинающим программистам и просто любознательным пользователям интересно, сколько в 1 байте битов. В современных компьютерах один байт всегда равняется восьми битам.

Если бит способен принимать только два значения, то сочетание восьми битов способно создавать 256 различных комбинаций. Число 256 образуется возведением двойки в восьмую степень (в соответствии с тем, сколько битов в байте).

Один бит — это 1 или 0. Два бита уже могут создавать комбинации: 00, 01, 10 и 11. Когда дело доходит до 8 бит, то вариантов сочетания нулей и единиц в диапазоне 00000000 … 11111111 получается как раз 256. Если запомнить, сколько значений может принимать и сколько бит содержится в одном байте, то запомнить эту цифру будет очень легко.

Каждое сочетание символов может нести в себе различную информацию в зависимости от кодировки (ASCII, Юникод и др.). Именно поэтому пользователи сталкиваются с тем, что введенная на русском языке информация иногда выводится в виде замысловатых символов.

Особенности двоичной системы счисления

Двоичная система имеет все те же свойства, что и привычная нам десятичная: числа, состоящие из единиц и нулей, можно складывать, вычитать, умножать и т. д. Разница лишь в том, что система состоит не из 10-ти, а всего из 2-х цифр. Именно поэтому ее удобно использовать для шифрования информации.

В любой позиционной числа состоят из разрядов: единиц, десятков, сотен и т. д. В десятичной системе максимальное значение одного разряда равно 9, а в бинарной системе — 1. Так как один разряд может принимать лишь два значения, бинарные числа быстро увеличиваются в длину. Например, привычное нам число 9 будет записано как 1001. Это значит, что девятка будет записана четырьмя символами, при этом один двоичный символ будет соответствовать одному биту.

Почему информация шифруется в двоичной форме?

Десятичная система удобна для ввода и вывода информации, а двоичная — для организации процесса ее преобразования. Также очень популярны системы, которые содержат восемь и шестнадцать символов: они переводят машинные коды в удобную форму.

Двоичная система наиболее удобна с точки зрения логики. Единица условно означает «да»: есть сигнал, утверждение истинно и т. д. Ноль ассоциируется со значением «нет»: значение ложно, сигнала нет и т. д. Любой открытый вопрос можно преобразовать в один или несколько вопросов с вариантами ответов «да» или «нет». Третий вариант, например, «неизвестно», будет абсолютно бесполезным.

В ходе развития компьютерных технологий были разработаны и трехразрядные емкости для хранения информации, которые называются триты. Они могут принимать три значения: 0 — емкость пуста, 1 — емкость заполнена наполовину и 2 — полная емкость. Однако двоичная система оказалась более простой и логичной, поэтому получила значительно большую популярность.

Сколько бит в байте было раньше?

Раньше нельзя было сказать однозначно, сколько бит в байте. Первоначально под байтом понимали машинное слово, то есть то количество бит, которое компьютер может обработать за один рабочий цикл (такт). Когда ЭВМ еще не помещались в рабочих кабинетах, разные микропроцессоры работали с байтами различных размеров. Байт мог включать в себя 6 бит, а у первых моделей IBM его размер достигал 9 бит.

Сегодня 8-битные байты стали настолько привычными, что даже в определении байта часто говорится, что это единица информации, состоящая из 8 бит. Тем не менее, в ряде архитектур байт равняется 32 битам и выступает в качестве машинного слова. Такие архитектуры применяются в некоторых суперкомпьютерах и сигнальных процессорах, но не на привычных нам компьютерах, ноутбуках и мобильных телефонах.

Почему победил восьмибитный стандарт?

Байты приобрели восьмибитный размер благодаря платформе с популярнейшим в свое время 8-битным процессором Intel 8086. Распространенность этой модели способствовала тому, что в 1970-х гг. 8 бит в байте фактически стало стандартным значением.

Восьмибитный стандарт удобен тем, что позволяет хранить в 1 байте два символа десятичной системы. При 6-битной системе возможно хранение одной цифры, в то время как 2 бита оказываются лишними. В 9 бит можно записать 2 цифры, но все равно остается один лишний бит. Число 8 является третьей степенью двойки, что обеспечивает дополнительное удобство.

Области использования битов и байтов

Многие пользователи задаются вопросом: как не перепутать бит и байт? В первую очередь необходимо обратить внимание на то, как написано обозначение: сокращенно байт пишется в виде большой буквы «Б» (на английском — «B»). Соответственно, для обозначения бита служит маленькая буква «б» («b»).

Однако всегда есть вероятность, что регистр выбран неверно (например, некоторые программы автоматически переводят весь текст в нижний или верхний регистр). В таком случае следует знать, что принято измерять в битах, а что — в байтах.

Традиционно байтами измеряют объемы: размер жесткого диска, флешки и любого другого носителя будет указан в байтах и укрупненных единицах, например, гигабайтах.

Биты служат для Количество информации, которую пропускает канал, скорость Интернета и т. п. измеряются в битах и производных единицах, например, мегабитах. Скорость скачивания файлов также всегда выводится в битах.

При желании можно перевести биты в байты или наоборот. Для этого достаточно вспомнить, сколько бит в байте, и произвести простое математическое вычисление. Биты превращаются в байты путем деления на восьмерку, обратный перевод осуществляется при помощи умножения на то же самое число.

Что такое машинное слово?

Машинное слово — это информация, записанная в ячейку памяти. Оно представляет собой максимальную последовательность единиц информации, которая обрабатывается, как одно целое.

Соответствует которая на протяжении длительного времени была равна 16 бит. В большинстве современных компьютеров она составляет 64 бита, хотя встречаются и более короткие (32 бита), и более длинные машинные слова. При этом число бит, образующих машинное слово, всегда кратно восьми и может быть легко переведено в байты.

Для конкретного компьютера длина слова является неизменной и относится к ряду важнейших характеристик «железа».

Для измерения длины есть такие единицы, как миллиметр, сантиметр, метр, километр. Известно, что масса измеряется в граммах, килограммах, центнерах и тоннах. Бег времени выражается в секундах, минутах, часах, днях, месяцах, годах, веках. Компьютер работает с информацией и для измерения ее объема также имеются соответствующие единицы измерения.

Мы уже знаем, что компьютер воспринимает всю информацию . Бит – это минимальная единица измерения информации, соответствующая одной двоичной цифре («0» или «1»).

Байт состоит из восьми бит. Используя один байт, можно закодировать один символ из 256 возможных (256 = 2 8). Таким образом, один байт равен одному символу, то есть 8 битам:

1 символ = 8 битам = 1 байту.

Изучение компьютерной грамотности предполагает рассмотрение и других, более крупных единиц измерения информации.

Таблица байтов:

1 байт = 8 бит

1 Кб (1 Килобайт ) = 2 10 байт = 2*2*2*2*2*2*2*2*2*2 байт =
= 1024 байт (примерно 1 тысяча байт – 10 3 байт)

1 Мб (1 Мегабайт ) = 2 20 байт = 1024 килобайт (примерно 1 миллион байт – 10 6 байт)

1 Гб (1 Гигабайт ) = 2 30 байт = 1024 мегабайт (примерно 1 миллиард байт – 10 9 байт)

1 Тб (1 Терабайт ) = 2 40 байт = 1024 гигабайт (примерно 10 12 байт). Терабайт иногда называют тонна .

1 Пб (1 Петабайт ) = 2 50 байт = 1024 терабайт (примерно 10 15 байт).

1 Эксабайт = 2 60 байт = 1024 петабайт (примерно 10 18 байт).

1 Зеттабайт = 2 70 байт = 1024 эксабайт (примерно 10 21 байт).

1 Йоттабайт = 2 80 байт = 1024 зеттабайт (примерно 10 24 байт).

В приведенной выше таблице степени двойки (2 10 , 2 20 , 2 30 и т.д.) являются точными значениями килобайт, мегабайт, гигабайт. А вот степени числа 10 (точнее, 10 3 , 10 6 , 10 9 и т.п.) будут уже приблизительными значениями, округленными в сторону уменьшения. Таким образом, 2 10 = 1024 байта представляет точное значение килобайта, а 10 3 = 1000 байт является приблизительным значением килобайта.

Такое приближение (или округление) вполне допустимо и является общепринятым.

Ниже приводится таблица байтов с английскими сокращениями (в левой колонке):

1 Kb ~ 10 3 b = 10*10*10 b= 1000 b – килобайт

1 Mb ~ 10 6 b = 10*10*10*10*10*10 b = 1 000 000 b – мегабайт

1 Gb ~ 10 9 b – гигабайт

1 Tb ~ 10 12 b – терабайт

1 Pb ~ 10 15 b – петабайт

1 Eb ~ 10 18 b – эксабайт

1 Zb ~ 10 21 b – зеттабайт

1 Yb ~ 10 24 b – йоттабайт

Выше в правой колонке приведены так называемые «десятичные приставки», которые используются не только с байтами, но и в других областях человеческой деятельности. Например, приставка «кило» в слове «килобайт» означает тысячу байт, также как в случае с километром она соответствует тысяче метров, а в примере с килограммом она равна тысяче грамм.

Возникает вопрос: есть ли продолжение у таблицы байтов? В математике есть понятие бесконечности, которое обозначается как перевернутая восьмерка: ∞.

Понятно, что в таблице байтов можно и дальше добавлять нули, а точнее, степени к числу 10 таким образом: 10 27 , 10 30 , 10 33 и так до бесконечности. Но зачем это надо? В принципе, пока хватает терабайт и петабайт. В будущем, возможно, уже мало будет и йоттабайта.

Напоследок парочка примеров по устройствам, на которые можно записать терабайты и гигабайты информации.

Есть удобный «терабайтник» – внешний жесткий диск, который подключается через порт USB к компьютеру. На него можно записать терабайт информации. Особенно удобно для ноутбуков (где смена жесткого диска бывает проблематична) и для резервного копирования информации. Лучше заранее делать резервные копии информации, а не после того, как все пропало.

Флешки бывают 1 Гб, 2 Гб, 4 Гб, 8 Гб, 16 Гб, 32 Гб, 64 Гб и даже 1 терабайт.

Могут вмещать 650 Мб, 700 Мб, 800 Мб и 900 Мб.

DVD-диски рассчитаны на большее количество информации: 4.7 Гб, 8.5 Гб, 9.4 Гб и 17 Гб.

Для измерения длины есть такие единицы, как миллиметр, сантиметр, метр, километр. Известно, что масса измеряется в граммах, килограммах, центнерах и тоннах. Бег времени выражается в секундах, минутах, часах, днях, месяцах, годах, веках. Компьютер работает с информацией и для измерения ее объема также имеются соответствующие единицы измерения.

Мы уже знаем, что компьютер воспринимает всю информацию через нули и единички. Бит — это минимальная единица измерения информации, соответствующая одной двоичной цифре («0» или «1»).

Байт состоит из восьми бит. Используя один байт, можно закодировать один символ из 256 возможных (256 = 28). Таким образом, один байт равен одному символу, то есть 8 битам:

1 символ = 8 битам = 1 байту.

Изучение компьютерной грамотности предполагает рассмотрение и других, более крупных единиц измерения информации.

Таблица байтов: 1 байт = 8 бит

1 Кб (1 Килобайт) = 210 байт = 2*2*2*2*2*2*2*2*2*2 байт =

1024 байт (примерно 1 тысяча байт — 103 байт)

1 Мб (1 Мегабайт) = 220 байт = 1024 килобайт (примерно 1 миллион байт — 106байт)

1 Гб (1 Гигабайт) = 230 байт = 1024 мегабайт (примерно 1 миллиард байт — 109байт)

1 Тб (1 Терабайт) = 240 байт = 1024 гигабайт (примерно 1012 байт). Терабайт иногда называют тонна.

1 Пб (1 Петабайт) = 250 байт = 1024 терабайт (примерно 1015 байт).

1 Эксабайт = 260 байт = 1024 петабайт (примерно 1018 байт).

1 Зеттабайт = 270 байт = 1024 эксабайт (примерно 1021 байт).

1 Йоттабайт = 280 байт = 1024 зеттабайт (примерно 1024 байт).


В приведенной выше таблице степени двойки (2 10 , 2 20 , 2 30 и т.д.) являются точными значениями килобайт, мегабайт, гигабайт.

Возникает вопрос: есть ли продолжение у таблицы байтов? В математике есть понятие бесконечности, которое обозначается как перевернутая восьмерка: ∞.

Понятно, что в таблице байтов можно и дальше добавлять нули, а точнее, степени к числу 10 таким образом: 10 27 , 10 30 , 10 33 и так до бесконечности. Но зачем это надо? В принципе, пока хватает терабайт и петабайт. В будущем, возможно, уже мало будет и йоттабайта.

Напоследок парочка примеров по устройствам, на которые можно записать терабайты и гигабайты информации. Есть удобный «терабайтник» — внешний жесткий диск, который подключается через порт USB к компьютеру. На него можно записать терабайт информации. Особенно удобно для ноутбуков (где смена жесткого диска бывает проблематична) и для резервного копирования информации. Лучше заранее делать резервные копии информации, а не после того, как все пропало.

Упражнения по компьютерной грамотности:

1) Сколько байт (без кавычек) содержит фраза «Сегодня 7 июля 2011 г.»?

2) Сколько байт (килобайт) занимает одна страница текста, если в одной строке помещается 60 символов, а на странице — 40 строк? Каков объем одной книги, состоящей из 100 подобных страниц?

3) Терабайтник — это внешний жесткий диск, который подключается к компьютеру через разъем USB, и имеет емкость 1 террабайт. В инструкции по его применению написано, что на этот диск может поместиться 250 тыс. музыкальных файлов или 285 тыс. фотографий. Каковы по мнению производителей этого устройства размер одного музыкального файла и размер одной фотографии?

4) Сколько подобных музыкальных файлов может поместиться на одном CD-диске размером 700 мегабайт?

5) Сколько подобных фотографий может поместиться на флешке размером 4 гигабайта?

Решения:

1) «Сегодня » — с пробелом (но без кавычек) 8 байт «7 июля » — с двумя пробелами (без кавычек) 7 байт «2010 г.» — с пробелом и с точкой (без кавычек) 7 байт Итого: 8 + 7 + 7 = 22 байта «весит» фраза «Сегодня 7 июля 2010 г.»

2) В одной строке помещается 60 символов, значит, объём одной строки 60 байт. На странице 40 таких строк, в каждой из которых содержится по 60 байт, поэтому объём одной страницы текста 60 x 40 = 2400 байт = 2,4 Килобайта = 2,4 Кб

Объём одной книги 2400 x 100 = 240 000 байт = 240 Килобайт = 240 Кб

3) Размер одного музыкального файла, который по мнению производителей можно записать на «терабайтник»: 1 000 000 000 000: 250 000 = (сокращаем по три нуля в делимом и в делителе) 1000 000 000: 250 = 4 000 000 байт = 4 Мегабайта = 4 Мб

Размер одной фотографии, который по мнению производителей можно записать на «терабайтник»: 1 000 000 000 000: 285 000 = (сокращаем по три нуля в делимом и в делителе) 1 000 000 000: 285 = 3 508 771, 93 байта = (округляем) 3,5 Мегабайта = 3,5 Мб

4) На CD-диске размером 700 мегабайт может поместиться 700 Мб: 4 Мб = 175 музыкальных файлов, каждый из которых размером не более 4 Мб. Здесь мегабайты можно сразу делить на мегабайты, а вот при работе с разными объёмами байтов лучше сначала переводить все в байты, а потом выполнять с ними различные арифметические операции.

5) На флешке размером 4 гигабайта может поместиться 4 000 000 000: 3 508 771, 93 = (сокращаем по три нуля в делимом и в делителе) = 4 000 000: 3 508 = 1 139,99 фото = (округляем) 1 140 фото, каждое из которых размером не более 3,5 Мб.

Можно считать и приблизительно. Тогда: На флешке размером 4 гигабайта может поместиться 4 000 000 000: 3 500 000 = (сокращаем по пять нулей в делимом и в делителе) = 40 000: 35 = 1 142,86 фото = (округляем в сторону уменьшения) 1 140 фото, каждое из которых размером не более 3,5 Мб

Сколько бит содержит 1 байт, и почему? | Рутвет

Оглавление:

  1. Хранение данных в компьютерной памяти
  2. Что такое бит?
  3. Что такое байт?
  4. Особенности двоичной системы исчисления
  5. Почему информацию шифруют в двоичном коде?
  6. Всегда ли байт содержал 8 бит?
  7. Почему победил именно 8-битный стандарт?
  8. Где применяются биты с байтами?

Абсолютно любая информация, заложенная в компьютере и на любых его носителях или периферийных устройствах, будь то стартовая программа BIOS или любые текстовые и графические документы, хранятся в его памяти в виде битов и байтов. Поэтому людей, пытающихся понять принцип работы компьютера, очень интересуют вопросы, касающиеся этих мельчайших элементов информации, а также, например, то, сколько бит в 1 байте информации содержится.

Хранение данных в компьютерной памяти

Компьютерная память – это невообразимо большое число ячеек, заполненных лишь единицами и нулями. Ячейкой называется минимальное звено диска, к которому считывающее устройство способно обратиться. В современных компьютерах она физически совпадает с триггером, который настолько мал, что под обычным оптическим микроскопом его увидеть почти невозможно. Каждая ячейка имеет свой уникальный адрес, по нему к ней обращаются любые программы.

Чаще всего ячейка совпадает с одним байтом. Но, поскольку архитектура компьютера может иметь разную разрядность, в ячейке может умещаться 2,4 и 8 байт. Электронные устройства воспринимают байт как мельчайшую единицу информации, хотя на самом деле он ещё делится на более элементарные ячейки – биты. Если в байте может быть закодирован единичный символ – цифра или буква, то в один бит они не «влезут». Хотя технически допустимо оперирование контроллеров единичными битами, но практически это почти не используется. Обычно происходит обращение либо к целым байтам, либо к их группам.

Что такое бит?

Чтобы понять, скольким битам равен 1 байт, нужно понять, что представляет собой бит. Часто битом называют мельчайшую единицу информации, но это определение не слишком точное, поскольку достаточно размыто само понятие информации. Более точно выглядит формулировка, определяющая бит, как букву компьютерного алфавита. Сам термин «бит» является сокращением английского словосочетания «binary digit», что в переводе на русский означает двоичная цифра. Компьютерный алфавит до невозможности прост, поскольку включает в себя лишь два символа – 0 и 1, что выражается как отсутствие или наличие сигнала или ложь и истина. С помощью этого простейшего набора логически можно описать абсолютно всё. Не более чем миф третье состояние компьютера – молчание, когда он не передаёт сигналы.

С точки зрения информации сам по себе символ не имеет никакой ценности, поскольку при виде нуля или единицы совершенно невозможно понять, к какого рода информации данное значение может относиться. И независимо от того, 1 байт сколько бит включает, любые программы, тексты и картинки состоят лишь из нулей и единиц. Поэтому в качестве самостоятельной единицы бит оказался не слишком удобен. Поэтому для кодирования удобоваримой информации биты потребовалось объединить в байты.

Что такое байт?

Если бит содержит в себе букву компьютерного алфавита, то байт можно сравнить со словом. В одном байте может содержаться целое число или часть большого числа, текстовый символ, два маленьких числа и прочее. То есть в нём уже присутствует минимальный объём осмысленной информации.

Многие любознательные пользователи и начинающие программисты интересуются тем, сколько бит содержит 1 байт.

В современных компьютерах это всегда 8 бит. Но если бит может иметь только 2 значения, то 8 битов байта дают уже 256 различных вариантов (два в восьмой степени даёт число 256).

Например, один бит даёт значения 0 или 1. Два бита уже позволяют комбинации: 00, 01, 10 и 11. Если же используется 8 бит, то в диапазоне между 00000000 и 11111111 помещается именно 256 значений. Не так сложно и запомнить число битов в байте, и сколько значений может принимать каждый байт. В зависимости от кодировки (Юникод, ASCII и прочих) каждое сочетание несёт в себе ту или иную информацию.  По этой причине попытка ввести данные на русском языке приводит к их выводу в виде своеобразных символов.

Особенности двоичной системы исчисления

Двоичная система позволяет все те же манипуляции с числами, что и классическая десятичная система: составленные из нулей и единиц числа можно складывать, умножать, делить и вычитать. Но при этом вся математика здесь обходится двумя цифрами, из-за чего она гораздо удобнее для шифрования информации. Любая позиционная система исчисления имеет разряды для чисел: единицы, десятки, сотни и т.д. Но если в десятичной системе максимальная величина одного разряда равна 9, то в бинарной системе это 1. Но поскольку в бинарном разряде есть лишь два значения, то длина бинарных числе очень быстро возрастает. Например, число 9 там выразится как 1001, то есть потребуется 4 символа, при этом каждый двоичный символ будет занимать один бит.

Почему информацию шифруют в двоичном коде?

Десятичная кодировка более удобна для ввода и вывода данных, зато двоичная облегчает процесс её преобразования. Есть ещё и другие системы, основанные на 8 и 16 символах, которые используются для переводов машинных кодов в приемлемую форму. С точки зрения логики двоичная система идеальна. Условно единице присвоено значение «да» или истинности, а ноль в противовес означает «нет» и ложь. Любой прямой вопрос можно разложить на несколько более простых вопросов, имеющих ответы «да» и «нет». А третий вариант («неизвестно») окажется совершенно избыточным. Исследования в области вычислительной техники привели к изобретению трёхразрядных единиц хранения данных, которые получили название тритов.

Диапазон их значений следующий:

  • 0 означает пустую ёмкость;
  • 1 наполовину заполненная ёмкость;
  • 2 полная ёмкость.

Но двоичная система оказалась более гибкой и логичной, поэтому и легла в основу компьютерной логики.

Видео о том, сколько бит в 1 байте информации

Всегда ли байт содержал 8 бит?

Ответ на вопрос, сколько битов содержится в 1 байте, не всегда был одинаковым, а когда-то он и вовсе не имел точного ответа. Под байтом первоначально понималось машинное слово – такое количество бит информации, которое ЭВМ могла обработать за один такт или рабочий цикл. Когда ЭВМ занимали целые залы, её логические схемы оперировали байтами разных размеров: у одних было 6 бит, а в первых моделях компьютеров IBM байт состоял из 9 бит. На данный момент практически победил стандарт байта из 8 бит, поэтому его даже называют единицей информации, включающей 8 бит. Но в некоторых архитектурах по-прежнему используются 32-битные байты, считающиеся за машинное слово. Подобная архитектура  используется в сигнальных процессорах и суперкомпьютерах, а во всех широко использующихся ноутбуках, компьютерах и мобильных устройствах только 8-битные.

Почему победил именно 8-битный стандарт?

8-битный стандарт байта стал доминантным из-за победы на рынке платформы IBMPC, которая использовала чрезвычайно популярный процессор Intel 8086. Благодаря её распространённости в 70-х годах прошлого века 8-битный байт стал фактически стандартом. Удобство 8-битного стандарта заключается в том, что в нём точно умещаются две цифры десятичной системы, в то время как 6-битная система способна вместить только один знак, при этом незаполненными останутся 2 бита. В 9-битный байт вписываются 2 знака, но остаётся неиспользованным один бит. К тому же восемь – это два в кубе, что также считается удобным.

Где применяются биты с байтами?

Неопытные пользователи часто путают обозначения бита и байта. Им нужно, в первую очередь, обратить внимание на написание обозначения. Сокращение байта использует заглавную букву «Б» или «B» в английском варианте, а более мелкому биту достались соответственно строчные буквы «б» или «b». Правда есть вероятность, что перепутан регистр, а некоторые программы переводят весь текст автоматически в верхний или нижний регистры. Поэтому лучше просто разделить то, что обычно измеряется в байтах, а что – в битах.

В байтах традиционно выражаются объёмы: флешки, жёсткого диска, дискеты, CD и любого другого носителя информации, причём в более масштабных единицах (килобайтах, мегабайтах, гигабайтах и т.д.).

Битами же измеряют скорость или пропускную способность канала, например, линии Интернета, здесь также господствуют порядковые величины – мегабиты и т.д. Скорость скачивания файлов также отражается в битах, которые при желании можно перевести в байты – просто умножить известную величину на восемь. Наоборот, делением объёма в байтах на восемь можно получить биты, хотя практически это вряд ли кому необходимо.

А Вы часто используете информацию о битах и байтах в жизни, и где Вы её применяете? Расскажите об этом в комментариях.

Бит равно байт. Сколько мегабайт в гигабайте, бит в байте (или килобайте) и что это вообще такое за единицы измерения информации. Кодирование целых и действительных чисел

Что такое Бит и Байт?

Давайте разберемся, что же такое бит и байт. Бит, наименьшая единица, которая измеряет количество информации. Один содержит мало информации в отличие от группы битов. В компьютере все данные хранятся и обрабатываются в виде знаков. Обычно используются всего два знака – цифры 0 и 1. Совокупность этих двух цифр называется двоичный код, а сами цифры принято называть двоичными цифрами или коротко битами. Компьютер различает 0 и 1 благодаря электрическим импульсам в электронных цепях. Если в цепи нет импульса – это цифра 0, если импульс есть, то это 1. Таким образом, в виде комбинации 0 и 1, внутри компьютера хранится абсолютно вся информация от фотографий до музыки. Наравне с понятием бита используется понятие байт.

Совокупность компьютерных данных из 8 бит называется байтом. 8 битов дают основу для представления символов, например буквы «А» и двоичной арифметики. То есть байт является командой битов, отвечающих за определенную деталь в файле. Каждый байт имеет в памяти компьютера уникальный адрес. По соглашению биты, и байты имеют нумерацию от 0 до 7 справа налево. Например: номер бита – 76543210, а значение его — 0 1 0 0 0 0 0 1 и в итоге, если передать это значение на принтер, там будет сгенерирована буква «А». Количество включенных битов в байте должно быть нечетно. Когда команда обращена к байту, компьютер проверяет этот байт и если число включенных битов четное, система выдает ошибку. Ошибка четности может быть результатом сбоя оборудования или случайным явлением, но это происходит очень редко.

Во время обработки данных, в компьютере по электронным цепям проходят электрические импульсы. Цепи состоят из проводников и электронных микро устройств, которые называются логическими вентилями. Импульсы, проходящие через эти вентили, могут «гаситься». Таким образом, обрабатываются данные. Объединяя логические вентили, создаются сложные комбинации, выполняющие операции – запоминают, сравнивают, складывают, сравнивают числа и прочее.

В кремниевых пластинках расположены электронные цепи. Каждая микросхема может содержать более миллиона цепей, от расположения зависит вид работы, которую они выполняют.
Микросхемы расположены на специальных пластинках, а именно на печатных платах. На самой плате напечатаны полоски, через которые проходит электричество к микросхемам. Металлические дорожки, которые называются шинами, передают байты, каждая шина содержит несколько таких дорожек. Одна дорожка передает один байт.

Шины делятся на три типа: шина данных, управления и адресная шина. Шина данных обменивается данными между процессором и устройствами ввода, между процессором и памятью компьютера. Инструкции от процессора ко всем узлам компьютера передаются по шине управления. С помощью адресной шины передается информация о местоположении или адресе данных.

Бит и байт это довольно маленькие величины, поэтому их используют с приставками кило, мега и гига. Давайте теперь поговорим о величине, которая измеряет скорость интернета. Скорость интернета, это количество отправляемой и получаемой информации вашим персональным компьютером в единицу времени. Качество единицы времени — секунда, а качество количества получаемой информации – килобит или мегабит. Например, если ваша скорость показывает 128 Kbps, значит, что ваше соединение пропускает 128 килобит что приравнивается к 16 килобайтам. Для того что бы узнать много это или мало воспользуйтесь тестами для определения скорости соединения с Интернетом.

Все фотографии, текстовые документы и программы хранятся в компьютерной памяти в виде битов и байтов. Что представляют собой эти мельчайшие единицы информации и сколько бит в байте?

Хранение данных в памяти

Компьютерная память представляет собой огромный набор ячеек, наполненных нулями и единицами. Ячейка — это минимальный объем данных, к которому может обращаться считывающее устройство. Физически она представляет собой триггер (в современных компьютерах). Триггер настолько мал, что его сложно рассмотреть даже под микроскопом. У каждой ячейки есть уникальный адрес, по которому ее находит та или иная программа.

Под ячейкой в большинстве случаев понимают один байт. Но, в зависимости от разрядности архитектуры, она может объединять в себе 2, 4 или 8 байт. Байт воспринимается электронными устройствами как единое целое, но на самом деле он состоит из еще меньших ячеек — битов. В 1 байте можно закодировать какой-нибудь символ, например, букву или цифру, в то время как 1 бита для этого недостаточно.

Контроллеры редко оперируют отдельными битами, хотя технически это возможно. Вместо этого идет обращение к целым байтам или даже группам байтов.

Что такое бит?

Часто под битом понимают единицу измерения информации. Такое определение нельзя назвать точным, потому что само понятие информации достаточно размыто. Если говорить более корректно, то бит — это буква компьютерного алфавита. Слово «бит» происходит от английского выражения «binary digit», что дословно означает «двоичная цифра».

Алфавит компьютеров прост и состоит всего из двух символов: 1 и 0 (наличие или отсутствие сигнала, истина или ложь). Этого набора вполне достаточно, чтобы логически описать все, что угодно. Третье состояние, под которым понимают молчание компьютера (прекращение передачи сигналов), является мифом.

Сама по себе буква не несет в себе никакой ценности с точки зрения информации: глядя на единицу или ноль, невозможно понять даже то, к какого рода данным это значение относится. И фото, и тексты, и программы в конечном счете состоят из единиц и нулей. Поэтому бит неудобен в качестве самостоятельной единицы. Следовательно, биты необходимо объединять для того, чтобы кодировать с их помощью полезную информацию.

Что такое байт?

Если бит — это буква, то байт представляет собой подобие слова. Один байт может содержать текстовый символ, целое число, часть большого числа, два небольших числа и т. д. Таким образом, в байте уже содержится осмысленная информация, хоть и в небольшом объеме.

Начинающим программистам и просто любознательным пользователям интересно, сколько в 1 байте битов. В современных компьютерах один байт всегда равняется восьми битам.

Если бит способен принимать только два значения, то сочетание восьми битов способно создавать 256 различных комбинаций. Число 256 образуется возведением двойки в восьмую степень (в соответствии с тем, сколько битов в байте).

Один бит — это 1 или 0. Два бита уже могут создавать комбинации: 00, 01, 10 и 11. Когда дело доходит до 8 бит, то вариантов сочетания нулей и единиц в диапазоне 00000000 … 11111111 получается как раз 256. Если запомнить, сколько значений может принимать и сколько бит содержится в одном байте, то запомнить эту цифру будет очень легко.

Каждое сочетание символов может нести в себе различную информацию в зависимости от кодировки (ASCII, Юникод и др.). Именно поэтому пользователи сталкиваются с тем, что введенная на русском языке информация иногда выводится в виде замысловатых символов.

Особенности двоичной системы счисления

Двоичная система имеет все те же свойства, что и привычная нам десятичная: числа, состоящие из единиц и нулей, можно складывать, вычитать, умножать и т. д. Разница лишь в том, что система состоит не из 10-ти, а всего из 2-х цифр. Именно поэтому ее удобно использовать для шифрования информации.

В любой позиционной числа состоят из разрядов: единиц, десятков, сотен и т. д. В десятичной системе максимальное значение одного разряда равно 9, а в бинарной системе — 1. Так как один разряд может принимать лишь два значения, бинарные числа быстро увеличиваются в длину. Например, привычное нам число 9 будет записано как 1001. Это значит, что девятка будет записана четырьмя символами, при этом один двоичный символ будет соответствовать одному биту.

Почему информация шифруется в двоичной форме?

Десятичная система удобна для ввода и вывода информации, а двоичная — для организации процесса ее преобразования. Также очень популярны системы, которые содержат восемь и шестнадцать символов: они переводят машинные коды в удобную форму.

Двоичная система наиболее удобна с точки зрения логики. Единица условно означает «да»: есть сигнал, утверждение истинно и т. д. Ноль ассоциируется со значением «нет»: значение ложно, сигнала нет и т. д. Любой открытый вопрос можно преобразовать в один или несколько вопросов с вариантами ответов «да» или «нет». Третий вариант, например, «неизвестно», будет абсолютно бесполезным.

В ходе развития компьютерных технологий были разработаны и трехразрядные емкости для хранения информации, которые называются триты. Они могут принимать три значения: 0 — емкость пуста, 1 — емкость заполнена наполовину и 2 — полная емкость. Однако двоичная система оказалась более простой и логичной, поэтому получила значительно большую популярность.

Сколько бит в байте было раньше?

Раньше нельзя было сказать однозначно, сколько бит в байте. Первоначально под байтом понимали машинное слово, то есть то количество бит, которое компьютер может обработать за один рабочий цикл (такт). Когда ЭВМ еще не помещались в рабочих кабинетах, разные микропроцессоры работали с байтами различных размеров. Байт мог включать в себя 6 бит, а у первых моделей IBM его размер достигал 9 бит.

Сегодня 8-битные байты стали настолько привычными, что даже в определении байта часто говорится, что это единица информации, состоящая из 8 бит. Тем не менее, в ряде архитектур байт равняется 32 битам и выступает в качестве машинного слова. Такие архитектуры применяются в некоторых суперкомпьютерах и сигнальных процессорах, но не на привычных нам компьютерах, ноутбуках и мобильных телефонах.

Почему победил восьмибитный стандарт?

Байты приобрели восьмибитный размер благодаря платформе с популярнейшим в свое время 8-битным процессором Intel 8086. Распространенность этой модели способствовала тому, что в 1970-х гг. 8 бит в байте фактически стало стандартным значением.

Восьмибитный стандарт удобен тем, что позволяет хранить в 1 байте два символа десятичной системы. При 6-битной системе возможно хранение одной цифры, в то время как 2 бита оказываются лишними. В 9 бит можно записать 2 цифры, но все равно остается один лишний бит. Число 8 является третьей степенью двойки, что обеспечивает дополнительное удобство.

Области использования битов и байтов

Многие пользователи задаются вопросом: как не перепутать бит и байт? В первую очередь необходимо обратить внимание на то, как написано обозначение: сокращенно байт пишется в виде большой буквы «Б» (на английском — «B»). Соответственно, для обозначения бита служит маленькая буква «б» («b»).

Однако всегда есть вероятность, что регистр выбран неверно (например, некоторые программы автоматически переводят весь текст в нижний или верхний регистр). В таком случае следует знать, что принято измерять в битах, а что — в байтах.

Традиционно байтами измеряют объемы: размер жесткого диска, флешки и любого другого носителя будет указан в байтах и укрупненных единицах, например, гигабайтах.

Биты служат для Количество информации, которую пропускает канал, скорость Интернета и т. п. измеряются в битах и производных единицах, например, мегабитах. Скорость скачивания файлов также всегда выводится в битах.

При желании можно перевести биты в байты или наоборот. Для этого достаточно вспомнить, сколько бит в байте, и произвести простое математическое вычисление. Биты превращаются в байты путем деления на восьмерку, обратный перевод осуществляется при помощи умножения на то же самое число.

Что такое машинное слово?

Машинное слово — это информация, записанная в ячейку памяти. Оно представляет собой максимальную последовательность единиц информации, которая обрабатывается, как одно целое.

Соответствует которая на протяжении длительного времени была равна 16 бит. В большинстве современных компьютеров она составляет 64 бита, хотя встречаются и более короткие (32 бита), и более длинные машинные слова. При этом число бит, образующих машинное слово, всегда кратно восьми и может быть легко переведено в байты.

Для конкретного компьютера длина слова является неизменной и относится к ряду важнейших характеристик «железа».

ЕдиницаАббревиатураСколько
битб0 или 1бит
байтБ8 бит
килобиткбит (кб)1 000 бит
килобайтКБайт (KБ)1024 байта
мегабитмбит (мб)1 000 килобит
мегабайтМБайт (МБ)1024 килобайта
гигабитгбит (гб)1 000 мегабит
гигабайтГБайт (ГБ)1024 мегабайта
терабиттбит (тб)1 000 гигабит
терабайтТБайт (ТБ)1024 гигабайта

Байт (byte) — единица хранения и обработки цифровой информации. Чаще всего байт считается равным восьми битам, в этом случае он может принимать одно из 256 (2’8) различных значений. Для того, чтобы подчеркнуть, что имеется в виду восьмибитный байт, в описании сетевых протоколов используется термин «октет» (лат. octet).

Килобайт (кБ, Кбайт, КБ) м., скл. — единица измерения количества информации, равная в зависимости от контекста 1000 или 1024 (2’10) стандартным (8-битным) байтам. Применяется для указания объёма памяти в различных электронных устройствах.
1 килобайт (КБ) = 8 килобит (Кб)

Мегабайт (Мбайт, М, МБ) м., скл. — единица измерения количества информации, равная, в зависимости от контекста, 1 000 000 (10’6) или 1 048 576 (2’20) стандартным (8-битным) байтам.

Гигабайт (Гбайт, Г, ГБ) — кратная единица измерения количества информации, равная 2’30 стандартным (8-битным) байтам или 1024 мегабайтам. Применяется для указания объёма памяти в различных электронных устройствах.

Терабайт (Тбайт, ТБ) м., скл. — единица измерения количества информации, равная 1 099 511 627 776 (2’40) стандартным (8-битным) байтам или 1024 гигабайтам. Применяется для указания объёма памяти в различных электронных устройствах.

Петабайт (ПБайт, ПБ) м., скл. — единица измерения количества информации, равная 25’0 стандартным (8-битным) байтам или 1024 терабайтам. Применяется для указания объёма памяти в различных электронных устройствах.

Эксабайт (Эбайт, Э, ЭБ) — единица измерения количества информации, равная 26’0 стандартным (8-битным) байтам или 1024 петабайтам. Применяется для указания объёма памяти в различных электронных устройствах.

Зеттабайт (Збайт, З, ЗБ) — единица измерения количества информации, равная 27’0 стандартным (8-битным) байтам или 1024 эксабайтам. Применяется для указания объёма памяти в различных электронных устройствах.

Йоттабайт (Йбайт, Й, ЙБ) — единица измерения количества информации, равная 1024 стандартным (8-битным) байтам или 1000 зеттабайтам. Применяется для указания объёма памяти в различных электронных устройствах.

1 Йoттабайт можно представить как:

103 = 1 000 Зеттабайтов

106 = 1 000 000 Эксабайтов

109 = 1 000 000 000 Петабайтов

1012 = 1 000 000 000 000 Терабайтов

1015 = 1 000 000 000 000 000 Гигабайтов

1018 = 1 000 000 000 000 000 000 Мегабайтов

1021 = 1 000 000 000 000 000 000 000 Килобайтов

1024 = 1 000 000 000 000 000 000 000 000 Байтов

Конвертор величин байт, бит, килобит, килобайт, мегабит, мегабайт, гигабит, гигабайт, терабит, терабайт, петабит, петабайт, эксбит, эксбайт

7,2 терабайта на один размером с обычный DVD диск

Австралийские исследователи создали технологию, которая теоретически позволяет записывать 7,2 терабайта данных на один диск размером с обычный DVD. Об этом сообщает Nature News, а статья исследователей появилась в журнале Nature.

В современных DVD-приводах запись информации осуществляется при помощи лазерного луча, который выжигает на поверхности диска выемки. Новая технология работает похожим образом. Основное отличие в том, что вместо появления выемок на поверхности диска плавятся золотые наноштыри.

Столь высокой плотности записи информации ученым удалось добиться при помощи нескольких технических приемов. Во-первых, исследователи использовали лазеры нескольких цветов. Дело в том, что лучи определенной длины волны воздействуют только на штыри с определенным соотношением длины и толщины. Во-вторых, исследователи использовали лучи с различной поляризацией, которые действуют на штыри, ориентированные определенным образом.

Используя лучи разного цвета и разной поляризации, представляется возможным записывать информацию на одном и том же регионе диска несколько раз. Так, например, два вида поляризации и три цвета (то есть в общей сложности шесть возможных комбинаций) позволяют записать 1,6 терабайта данных на диск размером с DVD. Если добавить еще один вариант поляризации, то получится диск объемом 7,2 терабайта.

Чтобы считывать информацию, исследователи используют слабый луч лазера, который не расплавляет наноштыри. При этом на выходе получается читаемый сигнал: эмпирически установлено, что наноштыри «откликаются» на слабый лазер гораздо лучше, чем, например, сферические наночастицы, в которые штыри превращаются после расплава.

Слабой стороной новой технологии является то, что исследователи используют лазерные импульсы очень короткой длительности — порядка нескольких фемтосекунд. Подобные лазеры дороги и сложны в производстве. Ученые надеются, что дальнейшее развитие технологии позволит обойти это ограничение. Они рассчитывают, что промышленное использование их открытия начнется примерно в 2020-х годах. ♌

Ловим Золотую рыбку в Интернете

Единицы измерения количества информации Основные: 1 бит – 0 или 1 1 байт = 8 бит Производные: 1 Кбайт (Килобайт) = 1024 байт 1 Мбайт (Мегабайт) = 1024 Кбайт 1 Гбайт (Гигабайт) = 1024 Мбайт 1 Тбайт (Терабайт) = 1024 Гбайт Более крупные: 1 Петабайт, 1 Экзабайт, 1 Зетабайт, 1 Йотабайт

Бит Бит Байт Байт КилобайтКилобайт Мегабайт Мегабайт Гигабайт Гигабайт … в более мелкие единицы – УМНОЖАТЬ на… *8 *1024 *1024 *1024 /8 /1024 /1024 /1024 …в более крупные единицы – ДЕЛИТЬ на… Пример: 725 байт = 725 * 8 = 5800 бит 725 байт = 725 / 1024 = 0,7 Килобайт Перевод единиц измерения информации

1) Сколько бит в 1 Килобайте? 2) Сколько байт и бит в 5 Килобайтах? 3) Переведите 7200 бит в байты и Килобайты * 8 = 8192 бит 5 * 1024 = 5120 байт 5120 * 8 = бит 7200 / 8 = 900 байт 900 / 1024 = 0,88 Кбайт Решите задачи:

5) Переведите 1536 Мбайт в Килобайты и Гигабайты 0,5 * 1024 = 512 Килобайт 512 * 1024 = байт * 8 = бит 4) Переведите 0,5 Мегабайт в Килобайты, байты и биты 1536 * 1024 = Килобайт 1536 / 1024 = 1,5 Гигабайт

6) Во сколько раз 1 Килобит 1 Килобита? в 1024 раза (сравниваем приставки Кило- и Мега-) 1 Килобита? в 1024 раза (сравниваем приставки Кило- и Мега-)»> 1 Килобита? в 1024 раза (сравниваем приставки Кило- и Мега-)»> 1 Килобита? в 1024 раза (сравниваем приставки Кило- и Мега-)» title=»6) Во сколько раз 1 Килобит 1 Килобита? в 1024 раза (сравниваем приставки Кило- и Мега-)»> title=»6) Во сколько раз 1 Килобит 1 Килобита? в 1024 раза (сравниваем приставки Кило- и Мега-)»>

3 байта … 24 бита 200 байт … 0,25 Кбайт 150 байт … 1100 бит 100 Мбайт … 0,1 Гбайт бит … 1,5 Кбайт 3,5 Кбайт…3600 байт = title=»3 байта … 24 бита 200 байт … 0,25 Кбайт 150 байт … 1100 бит 100 Мбайт … 0,1 Гбайт 12288 бит … 1,5 Кбайт 3,5 Кбайт…3600 байт =

8) Петя Кнопкин купил новый фильм в формате Blu-Ray объемом 25 Гбайт, а на компьютере его друга Коли Мышкина на винчестере свободно еще байт. Сможет ли Коля записать фильм на свой винчестер? Решение: 1) Переведем большое число в Кб Мб Гб / 1024 / 1024 / 1024 = 30 Гбайт Фильм поместиться, т.к. 25

Любой человек, который хоть немного взаимодействовал с компьютерами, знаком с такими терминами как «Гигабайт», «Мегабайт» и другими.

Они обозначают объем физического носителя информации, типа флешки, жесткого диска или же объем любого файла, хранящегося на компьютере.
Проще говоря – эта величина обозначает, сколько мест на компьютере занимает любой файл, или же сколько в сумме носитель способен вместить информации.

Если вы читаете эту статью с целью перевода одной единицы измерения в другую, тогда рекомендую сразу воспользоваться бесплатным онлайн калькулятором в низу страницы.

Вводите в поле любой значение, выбираете из списка величину и калькулятор произведет преобразование.

Что такое байт, килобайт, мегабайт, гигабайт

Несколько десятков лет назад память компьютеров была небольшой, и составляла не более десятка бит или пары байтов. Хранить там можно было несколько формул, пару примеров или математических выражений.

Сейчас же объемы жестких дисков составляют по несколько терабайт, а размеры файлов исчисляются гигабайтами. Поэтому с ходом компьютерного прогресса появилась проблема в записи того, сколько памяти занимает документ.

Именно тогда и были придуманы другие величины, которые полностью выходили из термина «бит».

Иначе говоря, термины «байт» , «килобайт» , «мегабайт» и «гигабайт» — это универсальные единицы измерения объема информации, которые обозначают то, сколько места файлы занимают на жестком диске.

Как оно работает?

Все жесткие диски, SD-карты, флешки можно объединить под одним общим названием – физический носитель .

Говоря простым языком, все эти физические носители состоят из небольших ячеек для хранения информации.

В них посредством двоичного кода записываются данные, которые переносятся на него. Эти ячейки называются битами, и именно они является наименьшей величиной компьютерной информации.

Когда вы переносите информацию на носитель – она как бы записывается в этих ячейках памяти и начинает занимать место.

Собственно, объем файла и обозначает, сколько байтов будет задействовано при хранении определенного файла. В этом и заключается принцип обозначения объема.

Кроме того, данные, которые используются в системе временно записываются в особый участок памяти – оперативную .

Они присутствуют там до тех пор, пока необходимы, и после этого выгружаются. Данные туда записываются в точно такие же ячейки, поэтому RAM имеет свое обозначение объема, пусть и гораздо меньшее, чем жесткие диски.

Что больше – мегабит или мегабайт

Нередко на описании USB-портов материнской платы, а также в характеристиках к флеш-картам и другим переносным носителям указывается скорость передачи информации.

Она обозначается как Гб/сек или Мб/сек, однако не надо путать их – это вовсе не гигабайт/секунду и не мегабайт/секунду.

В данном случае так обозначаются другие единицы измерения – мегабиты и гигабиты.

С их помощью измеряется скорость передачи информации.

Эти величины намного меньше, чем мегабайты и гигабайты, и вычисляются они, в отличие от вышеназванных объемов, в десятичной системе счисления.

Один мегабит равен примерно миллиону бит. Один гигабит равен миллиарду бит информации.

Почти всегда эти обозначения можно увидеть в скоростях интернет-провайдеров.

Поэтому, если скорость вашей сети равна 100 Мбит/сек, то за одну секунду подключения на ваш компьютер поступит 1 000 000 * 100 бит информации.

Технологии интернет-соединения дают возможность предлагать пользователям уже не мегабитные, а гигабитные варианты подключения.

Стандарты портов USB 3.0 позволяют передавать информацию на скорости 5Гбит/сек, и это далеко не предел – ведь уже сейчас в материнских платах появляются разъемы более высоких и скоростных версий.

Стоит отметить, что вопрос о том, что больше: мегабит или мегабайт – некорректен и на него нельзя дать ответ.

Это разные величины, разные способы измерения. Они хоть и сопоставляются между собой, однако, никто этого не делает, поскольку это не имеет смысла и практической пользы.

Сколько мегабайт в гигабайте

Все большее выходит из меньшего. Так, группа из восьми ячеек бита создает одну большую ячейку байта, то есть 8 бит = 1 байт .

  • 1024 байт = 1 килобайт,
  • 1024 килобайт = 1 гигабайт,
  • 1024 гигабайт = 1 терабайт.

Большие объемы не используются в домашних ПК, поэтому говорить о них нет особого смысла.

У рядового пользователя сразу встанет закономерный вопрос – а почему расчеты и градация такая странная?

Не проще ли было сделать так, чтобы 10 бит равнялись 1 байту, а 1 гигабайт соответствовал 1000 мегабайт?

Да, действительно, это было бы гораздо проще. Однако, проще в привычной нам системе счисления.

Дело вот в чем. В реальном мире мы используем диапазон чисел от 0 до 9. Это называется десятичная система счисления. Но компьютеры думают по-другому: они знают только два числа – 0 и 1, то есть система их вычислений двоичная .

Эти числа, условно, обозначают «Да» или «Нет». В данном случае они показывают, заполнена ячейка хранения информации, или нет.

Не вдаваясь в математику, стоит сказать только о том, что при переводе чисел из понятной компьютеру двоичной системы в нашу, десятеричную, двойка возводится в определенную степень.

А в степени двойки нету чисел, кратных 10. Именно поэтому расчеты такие странные: 1 байт в данном случае равен 2 в 3 степени бит и так далее.

Таким образом градация осуществляется от двойки, и число тем больше, чем большее количество раз ее перемножают саму на себя.

Почему HDD в 1Гб не равен 1000 Мб

Исходя из объяснения выше, один гигабайт больше, чем тысяча мегабайт ровно на 24 единицы. Поэтому в характеристиках на жестких дисках пишут точно – сколько составляет их объем. Округлять эти величины также нельзя.

Соответственно, 8 гигабайт оперативной памяти составляет не 8000 мегабайт, а 8192.

Именно по этой же причине иногда при покупке носителя информации его объем составляет немного меньше, чем написано в характеристиках.

Ровного значения просто не может быть, поэтому нередко вместо обещанных десяти гигабайт обнаруживается девять.

Где используются эти величины?

Как уже было сказано выше – эти термины применяются в компьютерной IT-сфере.

Например, при обозначении вместительности HDD. Современные жесткие диски уже имеют емкость больше одного терабайта, и продолжают расширяться.

С флешкартами и другими переносными носителями все скромнее – их максимальный объем может достигать 128 гигабайт.

Этими же терминами обозначается объем файлов.

Разброс в этом плане гораздо больше, бывают случаи, когда объемный и большой пласт информации весит несколько гигабайт, или же текстовый файл, занимающий всего пару килобайт.

Еще интереснее дела обстоят с оперативной памятью компьютера.

Ее объем также измеряется в ячейках памяти, и сейчас многие профессиональные машины оборудованы несколькими плашками RAM, общий размер которых может достигать 128 гигабайт.

Это обусловлено тем, что на обработку информации необходимо все больше и больше ресурсов – и для того, чтобы программа работала стабильно, во временной памяти должно быть много места.

А есть ли больше?

Существуют ли величины больше, чем терабайт? Да, конечно, они есть.

  • 1024 терабайт – это 1 петабайт.
  • 1024 петабайта – 1 экзабайт.

Дело в том, что современные технологии еще не дошли до создания носителей и уж тем более файлов, объемом и размером хотя бы приближенным к этим величинам – поэтому в повседневной жизни они используются крайне редко.

Однако, они широко используются для компьютерных расчетов в науке и высоких технологиях.

С учетом того, насколько быстро сейчас идет технологический прогресс – не исключено, что через пару лет на прилавках появятся жесткие диски объемом в 1024 терабайт

Таблица перевода величин: бит, байт, Кб, Мб, Гб, Тб

Существует таблица всех величин, которые используются в современных жестких дисках, других носителях информации, а также файлах.

Она создана специально для удобства точного определения объемов информации и дана ниже. В нее включены только те единицы измерения, которые можно увидеть и применить в реальной жизни.

После терабайта измерение хоть и ведется, однако на уровне науки и высоких технологий, а не повседневной жизни.

Достаточно просто определить, сколько бит в секунду передается к вам на компьютер, полученное значение разделить на 8, и потом на 1024.

Например, на скорости 100 Мб/сек в одну секунду вам будет передаваться примерно 12 мегабайт информации.

Недостаток таблицы заключается в том, что по ней можно определить только ровные значения, встретить которые можно нечасто.

Для того, чтобы точно определить вес файла или объем жесткого диска, можно воспользоваться онлайн-конвертером, который представлен чуть ниже.

Онлайн-конвертер величин

Конечно, информации, представленной в таблице величин, недостаточно для комфортных расчетов.

Очень мало файлов, вес которых будет точно равен одному гигабайту или сотне мегабайт, и поэтому даже имея под рукой эту справочную информацию, будет тяжело просчитать, носитель какого объема нужен для того, чтобы полностью перенести большой документ.

Именно для этого на этом сайте и установлен онлайн-конвертер величин.

Работает он очень просто – вы указываете объем и величину, в которой он выражен. Далее вам нужно выбрать значение, в которое требуется перевести число – и конвертер выдаст вам точное значение.

Что такое бит и байт информации. Что такое бит и байт

octet ).

В большинстве вычислительных архитектур байт — это минимальный независимо адресуемый набор данных.

История

Название «байт» было впервые использовано в июне 1956 года В. Бухгольцем (англ. Werner Buchholz ) при проектировании первого суперкомпьютера IBM 7030 Stretch для пучка одновременно передаваемых в устройствах ввода-вывода битов числом от одного до шести. Позже, в рамках того же проекта, байт был расширен до восьми бит. Слово byte было выбрано как намеренно искажённое слово bite , произносящееся так же (англ. bite — «кусок»; «часть чего-либо, отделённая за один укус»; ср. также появившееся позже название для 4-битной единицы «ниббл » от англ. nibble — «покусывать»). Изменённое написание byte через y вместо i потребовалось, чтобы избежать смешения со словом «бит» (bit ) . В печати слово byte впервые появилось в июне 1959 года .

Обозначение

Использование русской прописной буквы «Б» для обозначения байта регламентирует Межгосударственный (СНГ) стандарт ГОСТ 8.417-2002 («Единицы величин») в «Приложении А» и Постановление Правительства РФ от 31 октября 2009 г. № 879. Кроме того, констатируется традиция использования приставок СИ вместе с наименованием «байт» для указания множителей, являющихся степенями двойки (1 Кбайт = 1024 байт , 1 Мбайт = 1024 Кбайт , 1 Гбайт = 1024 Мбайт и т. д., причём вместо строчной «к» используется прописная «К»), и упоминается, что подобное использование приставок СИ не является корректным. По ГОСТ IEC 60027-2-2015 строчная «к» соответствует 1000 и «Ки» — 1024, так 1 КиБ = 1024 Б, 1 кБ = 1000 Б.

Использование прописной буквы «Б» для обозначения байта соответствует требованиям ГОСТ и позволяет избежать путаницы между сокращениями от байт и бит. Запись со строчной буквой в виде «Кб» (Мб, Гб) для обозначения байта будет не соответствовать международному стандарту IEC (и локализованному по нему ГОСТ). Однако авторы орфографического словаря приводят строчную форму «б» (и «Кб», «Мб», «Гб») для байта, как не образованную от фамилии.

Следует учитывать, что в ГОСТ 8.417, кроме «бит», для бита нет однобуквенного обозначения, поэтому использование записи вроде «Мб» как синонима для «Мбит» не соответствует этому стандарту. Но в некоторых документах используется сокращение b для bit: IEEE 1541-2002, IEEE Std 260.1-2004, в нижнем регистре: ГОСТ Р МЭК 80000-13-2016, ГОСТ IEC 60027-2-2015.

В международном стандарте МЭК IEC 60027-2 от 2005 года для применения в электротехнической и электронной областях рекомендуются обозначения:

  • bit — для бита;
  • o, B — для октета , байта. Причём о — единственное указанное обозначение во французском языке.

Склонение

Кроме обычной формы родительного падежа множественного числа (байтов, килобайтов, битов) существует счётная форма «байт» , которая используется в сочетании с числительными: 8 байт , 16 килобайт . Счётная форма является разговорной. Точно так же, например, с килограммами: обычная форма родительного падежа употребляется, если нет числительного, а в сочетании с числительным могут быть варианты: 16 килограммов (стилистически нейтральная обычная форма) и 16 килограмм (разговорная счётная форма).

Все фотографии, текстовые документы и программы хранятся в компьютерной памяти в виде битов и байтов. Что представляют собой эти мельчайшие единицы информации и сколько бит в байте?

Хранение данных в памяти

Компьютерная память представляет собой огромный набор ячеек, наполненных нулями и единицами. Ячейка — это минимальный объем данных, к которому может обращаться считывающее устройство. Физически она представляет собой триггер (в современных компьютерах). Триггер настолько мал, что его сложно рассмотреть даже под микроскопом. У каждой ячейки есть уникальный адрес, по которому ее находит та или иная программа.

Под ячейкой в большинстве случаев понимают один байт. Но, в зависимости от разрядности архитектуры, она может объединять в себе 2, 4 или 8 байт. Байт воспринимается электронными устройствами как единое целое, но на самом деле он состоит из еще меньших ячеек — битов. В 1 байте можно закодировать какой-нибудь символ, например, букву или цифру, в то время как 1 бита для этого недостаточно.

Контроллеры редко оперируют отдельными битами, хотя технически это возможно. Вместо этого идет обращение к целым байтам или даже группам байтов.

Что такое бит?

Часто под битом понимают единицу измерения информации. Такое определение нельзя назвать точным, потому что само понятие информации достаточно размыто. Если говорить более корректно, то бит — это буква компьютерного алфавита. Слово «бит» происходит от английского выражения «binary digit», что дословно означает «двоичная цифра».

Алфавит компьютеров прост и состоит всего из двух символов: 1 и 0 (наличие или отсутствие сигнала, истина или ложь). Этого набора вполне достаточно, чтобы логически описать все, что угодно. Третье состояние, под которым понимают молчание компьютера (прекращение передачи сигналов), является мифом.

Сама по себе буква не несет в себе никакой ценности с точки зрения информации: глядя на единицу или ноль, невозможно понять даже то, к какого рода данным это значение относится. И фото, и тексты, и программы в конечном счете состоят из единиц и нулей. Поэтому бит неудобен в качестве самостоятельной единицы. Следовательно, биты необходимо объединять для того, чтобы кодировать с их помощью полезную информацию.

Что такое байт?

Если бит — это буква, то байт представляет собой подобие слова. Один байт может содержать текстовый символ, целое число, часть большого числа, два небольших числа и т. д. Таким образом, в байте уже содержится осмысленная информация, хоть и в небольшом объеме.

Начинающим программистам и просто любознательным пользователям интересно, сколько в 1 байте битов. В современных компьютерах один байт всегда равняется восьми битам.

Если бит способен принимать только два значения, то сочетание восьми битов способно создавать 256 различных комбинаций. Число 256 образуется возведением двойки в восьмую степень (в соответствии с тем, сколько битов в байте).

Один бит — это 1 или 0. Два бита уже могут создавать комбинации: 00, 01, 10 и 11. Когда дело доходит до 8 бит, то вариантов сочетания нулей и единиц в диапазоне 00000000 … 11111111 получается как раз 256. Если запомнить, сколько значений может принимать и сколько бит содержится в одном байте, то запомнить эту цифру будет очень легко.

Каждое сочетание символов может нести в себе различную информацию в зависимости от кодировки (ASCII, Юникод и др.). Именно поэтому пользователи сталкиваются с тем, что введенная на русском языке информация иногда выводится в виде замысловатых символов.

Особенности двоичной системы счисления

Двоичная система имеет все те же свойства, что и привычная нам десятичная: числа, состоящие из единиц и нулей, можно складывать, вычитать, умножать и т. д. Разница лишь в том, что система состоит не из 10-ти, а всего из 2-х цифр. Именно поэтому ее удобно использовать для шифрования информации.

В любой позиционной числа состоят из разрядов: единиц, десятков, сотен и т. д. В десятичной системе максимальное значение одного разряда равно 9, а в бинарной системе — 1. Так как один разряд может принимать лишь два значения, бинарные числа быстро увеличиваются в длину. Например, привычное нам число 9 будет записано как 1001. Это значит, что девятка будет записана четырьмя символами, при этом один двоичный символ будет соответствовать одному биту.

Почему информация шифруется в двоичной форме?

Десятичная система удобна для ввода и вывода информации, а двоичная — для организации процесса ее преобразования. Также очень популярны системы, которые содержат восемь и шестнадцать символов: они переводят машинные коды в удобную форму.

Двоичная система наиболее удобна с точки зрения логики. Единица условно означает «да»: есть сигнал, утверждение истинно и т. д. Ноль ассоциируется со значением «нет»: значение ложно, сигнала нет и т. д. Любой открытый вопрос можно преобразовать в один или несколько вопросов с вариантами ответов «да» или «нет». Третий вариант, например, «неизвестно», будет абсолютно бесполезным.

В ходе развития компьютерных технологий были разработаны и трехразрядные емкости для хранения информации, которые называются триты. Они могут принимать три значения: 0 — емкость пуста, 1 — емкость заполнена наполовину и 2 — полная емкость. Однако двоичная система оказалась более простой и логичной, поэтому получила значительно большую популярность.

Сколько бит в байте было раньше?

Раньше нельзя было сказать однозначно, сколько бит в байте. Первоначально под байтом понимали машинное слово, то есть то количество бит, которое компьютер может обработать за один рабочий цикл (такт). Когда ЭВМ еще не помещались в рабочих кабинетах, разные микропроцессоры работали с байтами различных размеров. Байт мог включать в себя 6 бит, а у первых моделей IBM его размер достигал 9 бит.

Сегодня 8-битные байты стали настолько привычными, что даже в определении байта часто говорится, что это единица информации, состоящая из 8 бит. Тем не менее, в ряде архитектур байт равняется 32 битам и выступает в качестве машинного слова. Такие архитектуры применяются в некоторых суперкомпьютерах и сигнальных процессорах, но не на привычных нам компьютерах, ноутбуках и мобильных телефонах.

Почему победил восьмибитный стандарт?

Байты приобрели восьмибитный размер благодаря платформе с популярнейшим в свое время 8-битным процессором Intel 8086. Распространенность этой модели способствовала тому, что в 1970-х гг. 8 бит в байте фактически стало стандартным значением.

Восьмибитный стандарт удобен тем, что позволяет хранить в 1 байте два символа десятичной системы. При 6-битной системе возможно хранение одной цифры, в то время как 2 бита оказываются лишними. В 9 бит можно записать 2 цифры, но все равно остается один лишний бит. Число 8 является третьей степенью двойки, что обеспечивает дополнительное удобство.

Области использования битов и байтов

Многие пользователи задаются вопросом: как не перепутать бит и байт? В первую очередь необходимо обратить внимание на то, как написано обозначение: сокращенно байт пишется в виде большой буквы «Б» (на английском — «B»). Соответственно, для обозначения бита служит маленькая буква «б» («b»).

Однако всегда есть вероятность, что регистр выбран неверно (например, некоторые программы автоматически переводят весь текст в нижний или верхний регистр). В таком случае следует знать, что принято измерять в битах, а что — в байтах.

Традиционно байтами измеряют объемы: размер жесткого диска, флешки и любого другого носителя будет указан в байтах и укрупненных единицах, например, гигабайтах.

Биты служат для Количество информации, которую пропускает канал, скорость Интернета и т. п. измеряются в битах и производных единицах, например, мегабитах. Скорость скачивания файлов также всегда выводится в битах.

При желании можно перевести биты в байты или наоборот. Для этого достаточно вспомнить, сколько бит в байте, и произвести простое математическое вычисление. Биты превращаются в байты путем деления на восьмерку, обратный перевод осуществляется при помощи умножения на то же самое число.

Что такое машинное слово?

Машинное слово — это информация, записанная в ячейку памяти. Оно представляет собой максимальную последовательность единиц информации, которая обрабатывается, как одно целое.

Соответствует которая на протяжении длительного времени была равна 16 бит. В большинстве современных компьютеров она составляет 64 бита, хотя встречаются и более короткие (32 бита), и более длинные машинные слова. При этом число бит, образующих машинное слово, всегда кратно восьми и может быть легко переведено в байты.

Для конкретного компьютера длина слова является неизменной и относится к ряду важнейших характеристик «железа».

Что ныне измеряется в битах и байтах? Среднестатистический пользователь ПК о таких единицах измерения информации практически забыл. И забыл бы полностью, если бы не путаница между килобитами и килобайтами, в которых измеряются скорости путешествия в сети Интернет. Между тем, еще со школы каждый знает, что приставка кило- множит исходный показатель на тысячу. Пытаясь разобраться, пользователь делит, умножает и окончательно путается в дебрях арифметики. Возьмемся же за ключевые понятия — бит и байт — и посмотрим, под каким соусом их едят.

Определение

Бит — минимальная единица измерения количества информации (подобно букве в лингвистике). В двоичной системе счисления бит равен одному разряду.

Байт — единица хранения и обработки цифровой информации, представляющая собой совокупность битов, которые система может обрабатывать одновременно (в лингвистике назвали бы словом).

Сравнение

В одном байте — 8 бит. Бит может принимать значение 0 или 1, байт — от 0 до 256. Когда речь идет о скорости передачи данных, значения бит/с и байт/с (Кбит/с и Кб/с соответственно) существенно отличаются. В килобитах считается скорость подключения, или количество полученной/переданной информации за единицу времени. В килобайтах обычно отображается скорость скачивания файлов. Таким образом, при скорости подключения 128 Кбит/с скорость скачивания (в идеальных условиях) будет 16 Кб/с, то есть документ размером 160 Кб загрузится за 10 с.

Выводы сайт

  1. Бит — минимальная единица измерения, байт — единица хранения и обработки цифровой информации
  2. В одном байте 8 бит
  3. При определении скорости подключения обычно оперируют битами, скорости скачивания файлов — байтами.

В сегодняшней статье мы займемся измерением информации. Все картинки, звуки и видео ролики, которые мы с вами видим на экранах мониторов, представляют собой не более чем цифры. И эти цифры можно измерить, и, сейчас, вы научитесь переводить мегабиты в мегабайты и мегабайты в гигабайты.

Если вам важно знать, сколько в 1 гб мб или сколько в 1 мб кб, то эта статья для вас. Чаще всего такие данные нужны программистам, оценивающим занимаемый их программами объем, но, иногда, не мешает и рядовым пользователям для оценки размера скачиваемых или хранимых данных.

Если вкратце, то достаточно знать это:

1 байт = 8 бит

1 килобайт = 1024 байта

1 мегабайт = 1024 килобайта

1 гигабайт = 1024 мегабайта

1 терабайт = 1024 гигабайта

Общепринятые сокращения: килобайт=кб, мегабайт=мб, гигабайт=гб.

Недавно я получил вопрос от моего читателя: «Что больше кб или мб?». Надеюсь, теперь, ответ на него знает каждый.

Единицы измерения информации в подробностях

В информационно мире применяется не привычная для нас, десятеричная система измерения, а двоичная. Это значит, что одна цифра может принимать значение не от 0 до 9, а от 0 до 1.

Простейшей единицей измерения информации является 1 бит, он может быть равен 0 или 1. Но эта величина очень мала для современного объема данных, поэтому используют биты редко. Чаще применяют байты, 1 байт равен 8 бит и может принимать значение от 0 до 15 (шестнадцатеричная система исчисления). Правда вместо чисел 10-15 применяются буквы от А до F.

Но и эти объемы данных невелики, поэтому применяются привычные всем приставки кило- (тысяча), мега-(миллион), гига-(миллиард).

Стоит отметить, что в инфомире, килобайт равен не 1000 байт, а 1024. И если вы хотите узнать, сколько килобайт в мегабайте, то вы тоже получите число 1024. На вопрос, сколько мегабайт в гигабайте вы услышите тот же ответ – 1024.

Определяется это также особенностью двоичной системы исчисления. Если, при использовании десятков, каждый новый разряд мы получаем умножением на 10 (1, 10, 100, 1000 и т.д.), то в двоичной системе новый разряд появляется после умножения на 2.

Это выглядит вот так:

2, 4, 8, 16, 32, 64, 128, 256, 512, 1024

Число, состоящее из 10 цифр двоичной системы, может иметь всего лишь 1024 значения. Это больше чем 1000, но ближе всего к привычной приставке кило-. Аналогичным образом применяются и мега- и гига и тера-.

Как известно, компьютер оперирует информацией, однако очевидно, что не так, как мы. Каким же образом и как измерить эту информацию? Что же такое информация? Давайте разбираться! Для тех, кому нужно перевести биты, байты, килобайты, мегабайты, гигабайты, терабайты, я сделал удобную «считалку», которую Вы сможете скачать в конце статьи.

Информация — это все то, что Вы могли видеть, слышать или читать. Объёмы информации постоянно растут, и с каждым днем все быстрее и быстрее, поэтому встает проблема её хранения и систематизации, чтобы потом можно было что-то легко найти. Человечество дошло от наскальных записей и пергамента до цифровых носителей информации, однако понимать устройства хранения информации становится все сложнее.

Уже не раз упоминалось то, что компьютер обрабатывает информацию с помощью , что она передается от устройства к устройству в системном блоке с помощью кабелей. Также Вы уже знаете, что есть , например, устройства ввода (клавиатура и мышь, к примеру), с помощью которых мы можем отдавать команды компьютеру, а значит они тоже передают некую информацию. Для этого они подключаются к . Мы уже научились подключать некоторые устройства. Ну и, наконец, обработанная информация используется нами. Например, она возвращается к нам через устройства вывода, примером чему служит изображение на мониторе. Все мы привыкли к информации как, например, буквы в книге, наши записи в ежедневнике. Тут все просто: информация хранится в книге в виде текста, а книга на полке в библиотеке. А как Вы могли уже прочитать, информация в компьютере хранится на носителях . Вот, например, жесткий диск (про него читаем ) в системном блоке (он на фото)

Мы можем прочитать только то, что на наклейке, да и то непонятно значение большинства надписей. Однако в этой небольшой железяке, которую можно положить в карман, могут храниться миллионы книг и документов, тысячи изображений, аудио- и видеозаписей. Каким же образом? Дело в том, что компьютер — это машина, по проводам течет ток, и компьютер не может воспринимать ту же книгу или окружающий мир как мы. Зато прекрасно может определить есть сигнал или нет, ну или же маленькое или большое напряжение соответственно. Таким образом, компьютер может воспринять информацию о наличии или отсутствии сигнала как «да» или «нет» или, в цифровом эквиваленте, 0 или 1. Таким образом мы имеем нехитрую систему из нуля и единицы, которая и называется двоичной , так как цифры всего две. Одна цифра (0 или 1) называется бит — это самая маленькая единица компьютерной информации . Её компьютер и может хранить и передавать. Однако это очень мало, как же хранить, к примеру, слова?

Что такое байт. Сколько бит в байте.

Вы, наверное, слыхали про азбуку Морзе, где комбинации длинных и коротких сигналов (точек и тире) расшифровывались в слова. А если взять комбинацию из 8 цифр, каждая из которых может быть единицей или нулем, то получим 256 комбинаций, чего хватит для отображения и цифр и букв, причем и не одного алфавита. И вот эти 8 бит называются байтом . Таким образом в байте 8 бит . Это необязательно держать в голове или учить наизусть, можно работать на компьютере и без таких знаний, но Вам все же придется оценивать размер информации. Мерять информацию битами и даже байтами сложновато, потому как объёмы информации гораздо больше.

Что такое килобайт, мегабайт и гигабайт. Как перевести килобайты в мегабайты и гигабайты в мегабайты.

В десятичной системе исчисления мы используем приставки, чтобы обозначить большое число. Например: приставка кило- означает, что указанное число надо умножить на тысячу. 1 килограмм = 1000 грамм. Но килобайт — это не тысяча байт , а 2 в степени 10, то есть 1024 байт, что не совсем корректно. К этому сложновато сперва привыкнуть, даже есть такой анекдот:

— Чем отличается программист от обычного человека?

— Программист думает, что килограмм колбасы — это 1024 грамма, а обычный человек думает, что килобайт — это 1000 байт.

Приставка мега- предполагает миллион, но мегабайт — это опять же 1024 килобайт или 1048576 байт. Как видите, мегабайт больше килобайта. Гигабайт — это 1024 мегабайт = 1048576 килобайт = 1073741824 байт. Терабайт — это 1024 гигабайт соответственно.

Название

Обозначение

Сколько было бы в десятичной системе

В двоичной

1

Килобайт

1024

Мегабайт

10 6 = 1 000 000

1 048 576

Гигабайт

10 9 = 1 000 000 000

1 073 741 824

Терабайт

10 12 = 1 000 000 000 000

1 099 511 627 776

Петабайт

1 125 899 906 842 624

Вот самые распространенные единицы измерения объёма информации. Чтобы перевести килобайты в мегабайты , надо разделить их на 1024, а чтобы перевести гигабайты в мегабайты надо их умножить на 1024. Было предложено для устранения путаницы использовать для двоичных приставок «би», но кибибайт и мебибайт звучат не очень приятно и непривычно, поэтому они пока не прижились.

Чтобы понять, что будет представлять собой привычная нам вещь в электронном виде (в плане объема), дам примерные цифры:

  • Содержимое печатного листа А4 — 100 килобайт
  • 1,5 часа фильма в невысоком (для современных масштабов) качестве — 1,5 гигабайта. В высоком может быть и 40 гигабайт.
  • Фото среднего качества — 1-1,5 мегабайт
  • Аудиозапись среднего качества 3-5 минут — 10 мегабайт
Из этой статьи Вы узнали:
  • Что такое информация
  • Как информация представлена в компьютере
  • Что такое бит
  • Что такое байт
  • Какие есть приставки для измерения больших величин информации
  • Как перевести килобайты в мегабайты
  • Сколько мегабайт в гигабайте и многое другое

Информатика

Кодирование текстовой информации

 

Зачем вообще нужны кодирование текстов и почему это так важно? Какие кодировки текста существуют и какие из них следует использовать?

У вас встречалась ситуация, когда вы получаете электронное письмо, но не можете его прочитать – вместо текста идут какие-то непонятные знаки? То же самое случается и в интернете – открываете страницу, а разобрать ничего не возможно. Причем заметьте, подобное происходит именно с русским текстом, с английским подобные проблемы маловероятны. Причина проблем – открытие файла в неверной кодировке.

Соответствие между набором символов и набором числовых значений называется кодировкой символа.

В процессе вывода символа на экран производится обратная операция – декодирование, т.е. преобразование кода символа в изображение.

Присвоенный каждому символу конкретный числовой код фиксируется в кодовых таблицах. Причем, в разных таблицах одному и тому же символу могут соответствовать разные числовые коды. Обычно перекодированием текста занимаются специальные программы-конвертеры, они встроены в большинство приложений.

 

Зачем нужны кодировки

Символы на экране вашего компьютера формируются на основе двух вещей — наборов векторных форм (представлений) всевозможных символов (они находятся в файлах со шрифтами, которые установлены на вашем компьютере) и кода, который позволяет выдернуть из этого набора векторных форм (файла шрифта) именно тот символ, который нужно будет вставить в нужное место.

Кодировка ASCII

Для начала немного посчитаем. Помните, что такое бит? Это минимальный носитель информации, ноль или один. А байт содержит восемь битов. Сколько может быть комбинаций из нулей и единиц длины 8? Ответ – 2*2*2*2*2*2*2*2=256. Именно столько значений может принимать один байт. Иногда еще байт называют символом – потому что как раз для кодировки символа и стали использовать один байт. Даже меньше, изначально была придумана кодировка ASCII, которая использовала 7 битов – в первые 128 значений можно было вольготно разместить английский алфавит в обоих регистрах, диакритические знаки, цифры и набор спецсимволов. И эта кодировка действительно стала универсальной, поэтому англоязычные пользователи крайне редко могут испытывать проблемы с кодировкой.

Кодировка ASCII (American Standard Code for Information Interchange, которая по-русски обычно произносится как «аски») описывает первые 128 символов из наиболее часто используемых англоязычными пользователями — латинские буквы, арабские цифры и знаки препинания. Так же еще в эти 128 символов кодировки ASCII попадали некоторые служебные символы, вроде скобок, решеток, звездочек и т.п. Именно эти 128 символов из первоначального вариант ASCII стали стандартом, и в любой другой кодировке текста вы их обязательно встретите и стоять они будут именно в таком порядке. Но дело в том, что с помощью одного байта информации можно закодировать не 128, а целых 256 различных значений (двойка в степени восемь равняется 256), поэтому вслед за базовой версией ASCII появился целый ряд расширенных кодировок ASCII, в которых можно было кроме 128 основных символов закодировать еще и символы национальной кодировки (например, русской).

 

Для представления текстовой информации в компьютере достаточно 256 различных знаков.

Воспользуемся формулой Хартли, чтобы вычислить количество информации для кодирования одного знака:

 

N=2i          256=28           i=8 битов

 

Значит, длина кода для одного символа составляет

8 двоичных знаков (0 или 1)

Рис.1 Стандартная часть кодовой таблицы

Кодировка КОИ-8R

Принцип работы кодировки KOI-8R такой— каждый символ текста кодируется одним единственным байтом.

Среди особенностей кодировки KOI-8R можно отметить то, что русские буквы в ее таблице идут не в алфавитном порядке. В кодировке KOI-8R русские буквы расположены в тех же ячейках таблицы, что и созвучные им буквы латинского алфавита из первой части таблицы ASCII. Это было сделано для удобства перехода с русских символов на латинские путем отбрасывания всего одного бита (два в седьмой степени или 128).

Рис.2 Национальная часть кодовой таблицы

 

Кодировка Unicode

Перейдем к кириллице. Для нее стали использовать вторую половину кодовой таблицы – символы 129-256. Однако так сложилось, что различные кодировки были изобретены независимо – одни изобретатели располагали буквы, стремясь к соответствию расположения на пишущих машинках, другие – к тому, чтобы одинаково выглядящие кириллические и латинские буквы находились на расстоянии в 128 (что приводило к тому, что даже программы, не умевшие работать со второй половиной таблицы, более-менее читаемо отображали русский текст). Были и другие идеи; появление Windows также принесло свою кодировку. Но главное, такая кодировка принципиально не могла стать универсальной, так как если латиница была нужна везде (хотя бы для командной строки), то кириллица была лишь одним из национальных алфавитов.

У единой, потенциально универсальной кодировки, существует название: Unicode, и придумана она была уже давно, в 1991 году. В Юникоде используется 6 байтов для отображения символа.

Например, в операционной системе Windows вы можете пройти по пути Пуск — Программы — Стандартные — Служебные — Таблица символов. В результате откроется таблица с векторными формами всех установленных у вас в системе шрифтов. Если вы выберите в Дополнительных параметрах набор символов Юникод, то сможете увидеть для каждого шрифта в отдельности весь ассортимент входящих в него символов. Кстати, щелкнув по любому из этих символов вы сможете увидеть его двухбайтовый код в кодировке UTF 16, состоящий из четырех шестнадцатеричных цифр.

 

По формуле Хартли определим количество символов, которые можно закодировать по этому стандарту:

 

N=2i          216=65536

 

Такого количества символов оказалось достаточно, чтобы закодировать русский, латинский, греческий, арабский, иврит и другие алфавиты, цифры, знаки и математические символы.

 

 

Кодировки кириллицы

Как правило, для хранения кода символа используется 1 байт (8 битов).

Поэтому коды символов могут принимать значение от 0 до 255. Такие кодировки называют однобайтными. Они позволяют использовать 256 символов (2 в восьмой степени равно 256). Таблица однобайтных кодов символов ASCII состоит из двух частей. Первая часть таблицы ASCII-кодов (от 0 до 127) стандартна для всех компьютеров и содержит:

·                                 коды управляющих символов,

·                                 коды цифр, арифметических операций, знаков препинания,

·                                 некоторые специальные символы,

·                                 коды больших и маленьких латинских букв.

Вторая часть таблицы (коды от 128 до 255) бывает различной в различных компьютерах. Она содержит:

·                                 коды букв национального алфавита,

·                                 коды некоторых математических символов,

·                                 коды символов псевдографики.

Для русских букв используются такие различные кодовые таблицы: КОИ-8, ISO, Mac, CP1251, CP866.

 

 

В последнее время широкое распространение получил новый международный стандарт Unicode.

В Unicode отводится по 2 байта (16 битов) для кодирования каждого символа.

Поэтому с его помощью можно закодировать 65536 различных символов (2 в шестнадцатой степени равно 65536). Коды символов могут принимать значение от 0 до 65535.

 

Примеры решения задач

Задача 1. С помощью кодировки Unicode закодирована следующая фраза: Я хочу поступить в университет!Оцените информационный объем этой фразы.

В данной фразе содержится 31 символ (включая пробелы и знак препинания). Поскольку в кодировке Unicode каждому символу отводится 2 байта памяти, для всей фразы понадобится 31*2 = 62 байта или 31*2*8 = 496 битов.

Задача 2. Статья, набранная на компьютере, содержит 8 страниц, на каждой странице 40 строк, в каждой строке 64 символа. В одном из представлений Unicode каждый символ кодируется 16 битами. Определите информационный объем статьи в этом варианте Unicode. Выберите верный ответ из предложенных: а) 320 байт, б) 35 Кбайт , в) 640 байт, г) 40 Кбайт.

Определим количество символов: 8*40*64 = 20480. Поскольку в кодировке Unicode каждому символу отводится 16 битов памяти, для всей фразы понадобится 20480*16 = 327680 битов.

Переведем полученное значение в байты. Т.к. 8 бит = 1 байт, то 327680 : 8 = 40960 байт

Теперь в Кбайты. Т.к. 1024 байт = 1 Кбайт, то 40960 : 1024 = 40 Кбайт.

Из предложенных вариантов подходит вариант г) 40 Кбайт.

Задача 3. Автоматическое устройство осуществило перекодировку информационного сообщения на русском языке, первоначально записанного в 16-битном коде Unicode, в 8-битную кодировку КОИ-8. При этом информационное сообщение уменьшилось на 800 бит. Какова длина сообщения в символах?

1) 50 2) 100 3) 400 4) 800

Изменение кодировки с 16 бит на 8 бит, равно 16 — 8 = 8 бит .Следовательно информационный объем каждого символа сообщения уменьшился на 8 бит. Так как объем информационного сообщения уменьшился на 800 бит, следовательно количество символов в сообщение равно 800/8=100.

Ответ: Длина сообщения — 100 символов.

 

Задача 4. Автоматическое устройство осуществило перекодировку информационного сообщения длиной 48 символов, первоначально записанного в 7–битном коде ASCII, в 16–битную кодировку Unicode.

При этом информационное сообщение увеличилось на

1) 48 байт 2) 96 байт 3) 54 байт 4) 432 байт

Изменение кодировки с 7 бит на 16 бит, равно 16 — 7 = 9 бит. Следовательно информационный объем каждого символа сообщения увеличился на 9 бит.Так как длина сообщения равна 48 символов, информационное сообщение увеличилось на 48 * 9 = 432 бита. В предложенных ответах данного ответа нет, следовательно переведем биты в байты — 432/8 = 54 байта.

Ответ: Информационное сообщение увеличилось на 54 байта.

Задача 5. Автоматическое устройство осуществило перекодировку информационного сообщения на русском языке, первоначально записанного в 16–битном коде Unicode, в 8–битную кодировку Windows–1251, при этом информационный объем сообщения составил 60 байт.

Определите информационный объем сообщения до перекодировки.

1) 60 бит 2) 120 бит 3) 960 бит 4) 60 байт

По условиям задачи информационный объем сообщения после кодировки составил 60 байт = 480 бит, следовательно количество символов в сообщении = 480/8 = 60. До перекодировки сообщение кодировалось 16-битным кодом, следовательно информационный объем сообщения до перекодировки = 60 * 16 = 960 бит.

Ответ: информационный объем сообщения до перекодировки 960 бит.

Задача 6. Автоматическое устройство осуществило перекодировку информационного сообщения, первоначально записанного в 7-битном коде ASCII, в 16-битную кодировку Unicode. При этом информационное сообщение увеличилось на 108 бит.

Какова длина сообщения в символах?

1) 12 2) 27 3) 6 4) 62

Изменение кодировки с 7 бит на 16 бит, равно 16 — 7 = 9 бит. Следовательно информационный объем каждого символа сообщения увеличился на 9 бит. По условиям задачи информационный объем сообщения после кодировки составил 108 бит, следовательно количество символов сообщения = 108/9 = 12.

Ответ: длина сообщения 12 символов.

 

Задача 7. В кодировке Unicode на каждый символ отводится два байта. Определите информационный объем слова из двадцати четырех символов в этой кодировке.

1) 384 бита 2) 192 бита 3) 256 бит 4) 48 бит

Решение задачи 5.

Количество символов в сообщении — 24, каждый символ кодируется 2 байтами = 16 бит, следовательно 24 * 16 = 384 бит.

Ответ: 384 бита.

 

Решите самостоятельно

 

  1. В кодировке КОИ-8 каждый символ кодируется 1 байтом. Определите информационный объем сообщения из 20 символов в этой кодировке. Варианты ответов: а) 20 бит, б) 80 бит, в) 160 бит, г) 320 бит.
  2. В кодировке Unicode на каждый символ отводится 2 байта. Определите информационный объем слова из 24 символов в этой кодировке. Найдите верный ответ: а) 384 бита, б) 192 бита, в) 256 бит, г) 48 бит.

Типы данных в ассемблере


 

Данные – числа и закодированные символы, используемые в качестве операндов команд.
Основные типы данных в ассемблере

ТипДирективаКоличество байт
БайтDB1
СловоDW2
Двойное словоDD4
8 байтDQ8
10 байтDT10

Данные, обрабатываемые вычислительной машиной, можно разделить на 4 группы:

  • целочисленные;
  • вещественные.
  • символьные;
  • логические;
Целочисленные данные

Целые числа в ассемблере могут быть представлены в 1-байтной, 2-байтной, 4-байтной или 8-байтной форме. Целочисленные данные могут представляться в знаковой и беззнаковой форме.

Беззнаковые целые числа представляются в виде последовательности битов в диапазоне от 0 до 2n-1, где n- количество занимаемых битов.


Знаковые целые числа представляются в диапазоне -2n-1 … +2n-1-1. При этом старший бит данного отводится под знак числа (0 соответствует положительному числу, 1 – отрицательному).

Вещественные данные

Вещественные данные могут быть 4, 8 или 10-байтными и обрабатываются математическим сопроцессором.

Логические данные

Логические данные представляют собой бит информации и могут записываться в виде последовательности битов. Каждый бит может принимать значение 0 (ЛОЖЬ) или 1 (ИСТИНА). Логические данные могут начинаться с любой позиции в байте.

Символьные данные

Символьные данные задаются в кодах и имеют длину, как правило, 1 байт (для кодировки ASCII) или 2 байта (для кодировки Unicode)  .

Числа в двоично-десятичном формате

В двоично-десятичном коде представляются беззнаковые целые числа, кодирующие цифры от 0 до 9. Числа в двоично-десятичном формате могут использоваться в одном из двух видов:

  • упакованном;
  • неупакованном.

В неупакованном виде в каждом байте хранится одна цифра, размещенная в младшей половине байта (биты 3…0).

Упакованный вид допускает хранение двух десятичных цифр в одном байте, причем старшая половина байта отводится под старший разряд.

Числовые константы

Числовые константы используются для обозначения арифметических операндов и адресов памяти. Для числовых констант в Ассемблере могут использоваться следующие числовые форматы.

Десятичный формат – допускает использование десятичных цифр от 0 до 9 и обозначается последней буквой d, которую можно не указывать, например, 125 или 125d. Ассемблер сам преобразует значения в десятичном формате в объектный шестнадцатеричный код и записывает байты в обратной последовательности для реализации прямой адресации.

a DB 12

Шестнадцатеричный формат – допускает использование шестнадцатеричных цифр от 0 до F и обозначается последней буквой h, например 7Dh. Так как ассемблер полагает, что с буквы начинаются идентификаторы, то первым символом шестнадцатеричной константы должна быть цифра от 0 до 9. Например, 0Eh.

a DB 0Ch

Двоичный формат – допускает использование цифр 0 и 1 и обозначается последней буквой b. Двоичный формат обычно используется для более четкого представления битовых значений в логических командах (AND, OR, XOR).

a DB 00001100b

Восьмеричный формат – допускает использование цифр от 0 до 7 и обозначается последней буквой q или o, например, 253q.

a DB 14q

Массивы и цепочки

Массивом называется последовательный набор однотипных данных, именованный одним идентификатором.

Цепочка — массив, имеющий фиксированный набор начальных значений.

Примеры инициализации цепочек

M1 DD 0,1,2,3,4,5,6,7,8,9
M2 DD 0,1,2,3

   DD 4,5,6,7

   DD 8,9

Каждая из записей выделяет десять последовательных 4-байтных ячеек памяти и записывает в них значения 0, 1, 2, 3, 4, 5, 6, 7, 8, 9.

Идентификатор M1 определяет смещение начала этой области в сегменте данных .DATA.

Для инициализации всех элементов массива одинаковыми значениями используется оператор DUP:

Идентификатор Тип Размер DUP (Значение)

Идентификатор — имя массива;
Тип — определяет количество байт, занимаемое одним элементом;
Размер — константа, характеризующая количество элементов в массиве
Значение — начальное значение элементов.

Например

a DD 20 DUP (0)

описывает массив a из 20 элементов, начальные значения которых равны 0.

Если необходимо выделить память, но не инициализировать ее, в качестве поля Значение используется знак ?. Например,

b DD 20 DUP(?)

Символьные строки

Символьные строки представляют собой набор символов для вывода на экран. Содержимое строки отмечается

  • одиночными кавычками », например, ‘строка’
  • двойными кавычками «», например «строка»

Символьная строка определяется только директивой DB, в которой указывается более одного символа в последовательности слева направо.

Символьная строка, предназначенная для корректного вывода, должна заканчиваться нуль-символом ‘\0’ с кодом, равным 0.

Str DB ‘Привет всем!’, 0

Для перевода строки могут использоваться символы

  • возврат каретки с кодом 13 (0Dh)
  • перевод строки с кодом 10 (0Ah).

Stroka DB «Привет», 13, 10, 0

Назад


Назад: Язык ассемблера

Двоичное слово — обзор

III Коды постоянного веса

A Код постоянного веса (CW) с параметрами n , d , w — это набор C двоичных слов длиной n все имеют вес w , так что расстояние между любыми двумя кодовыми словами составляет не менее d . Все нетривиальные ( n , d , w ) коды CW имеют d ≤ 2 w . Пусть A ( n , d , w ) будет наибольшим количеством кодовых слов в любом коде CW с этими параметрами.Тогда классическая проблема состоит в том, чтобы определить это число или найти наилучшие верхние и нижние границы для A ( n , d , w ).

Двоичные коды CW нашли применение в задачах синхронизации, в таких областях, как системы связи с оптическим кодированием и множественным доступом (CDMA), связь с расширенным спектром со скачкообразной перестройкой частоты, модельное радио, проектирование сигналов радаров и гидролокаторов, а также построение последовательностей протоколов. для многопользовательского канала коллизий без обратной связи.Коды с постоянным весом по сравнению с другими алфавитами привлекли некоторое внимание, но пока что было немного приложений. Мы будем обсуждать только двоичные коды CW.

Коды с постоянным весом были тщательно изучены, и хорошей ссылкой на них является MacWilliams and Sloane (1977). Эрик Рейнс и Нил Слоан ведут таблицу наиболее известных нижних границ для A ( n , d , w ) на веб-сайте: http://www.research.att.com/njas/ коды / Andw /. Мы представим обзор этой темы с акцентом на связи с дизайном.

Поскольку сумма любых двух двоичных слов с одинаковым весом всегда имеет четный вес, мы имеем A ( n , 2δ — 1, w ) = A ( n , 2δ, w ). С этого момента мы будем предполагать, что расстояние d четное. У нас также есть A ( n , d , w ) = A ( n , d , n w ), поскольку когда два слова находятся на расстоянии d отдельно, так же их дополнения.Это означает, что нужно рассматривать только случай w n /2.

Связь между кодами CW и проектами очевидна. С точки зрения наборов, код CW — это просто набор из w подмножеств из набора n , где пересечение любых двух подмножеств w содержит не более t = w − d2 элементов. Эквивалентно, CW-код — это частичная система Штейнера Sw − d2 + 1, w, n. Тогда мы имеем

A (n, d, w) ≤n (n − 1)… (n − w + d / 2) w (w − 1)… (d / 2)

с равенством тогда и только тогда, когда система Штейнера S (w − d2 + 1, w, n) существует.

Интерес к кодам CW также связан с проблемой поиска линейных (или нелинейных) кодов ( n , M , d ) максимального размера M . Очевидно, что A ( n , d , w ) является верхней границей количества слов данного веса в таком максимальном коде. И наоборот, такие коды (или их смежные классы) могут давать нижние границы для A ( n , d , w ). В частности, более сильная версия границы Хэмминга (приведенная в разделе о совершенных кодах) была первоначально доказана с использованием A ( n , 2 t + 2,2 t + 1).

A ( n , 2 t + 2,2 t + 1) — это просто количество блоков в максимальном частичном S ( t + 1,2 t + 1 , n ) конструкция или упаковка. Если C — это код с исправлением ошибок t , то для любого c C количество блоков в соседней упаковке ∣ NS ( c ) ∣ ≤ A ( n , 2 т + 2,2 т + 1).Количество слов, которые находятся на расстоянии t + 1 от c , но не на расстоянии t от любого другого кодового слова, равно

(nt + 1) — (2t + 1t + 1) | NS (c) | ≥ ( nt + 1) — (2t + 1t + 1) A (2t + 2,2t + 1).

Каждое такое слово находится на расстоянии t + 1 от не более ⌊ n / t + 1⌋ других кодовых слов. Таким образом, суммируя все c C , каждое такое слово считается не более указанного количества раз. Это дает более сильную версию границы Хэмминга:

| C | ((∑i = 0t (ni)) + (nt + 1) — (2t + 1t + 1) A (n, 2t + 2,2t + 1 ) ⌊N / (t + 1) ⌋) ≤2n.

Коды с постоянным весом не могут быть линейными, поскольку это означало бы, что нулевой вектор был в коде, но можно иметь код, в котором все ненулевые слова имеют одинаковый вес. Эти коды иногда называют линейными эквидистантными кодами . Двойник кода Хэмминга (также называемый симплексным кодом ) является примером такого кода. Фактически было доказано, что единственные такие коды формируются путем взятия нескольких копий симплексного кода. Доказательства того, что все такие коды являются обобщенными симплексными кодами, явно вытекают из теории кодирования (Bonisoli, 1983), а также неявно из результатов по планам и системам множеств (Teirlinck, 1980).Существует тесная связь между линейными эквидистантными кодами и конечной геометрией. Слова симплексного кода соответствуют гиперплоскостям проективного пространства [более GF (2)] так же, как слова веса 3 в коде Хэмминга соответствуют строкам в этом проективном пространстве. [О связях между кодами и конечной геометрией см. Black and Mullin (1976).]

Другой вариант кодов CW — это оптические ортогональные коды (OOC), которые были применены в оптических системах связи CDMA.Вкратце, an ( n , w , t a , t b ) OOC — это код CW, C , длиной n и весом w . такое, что для любого c = ( c 0 , c 1 ,…, c n −1 ) ∈ C , и каждое y C , c y и каждый i ≢ 0 (mod n ),

(1) ∑j = 0n − 1cjcj + i≤ta,

и

(2) ∑j = 0n −1cjyj + i≤tc.

Уравнение (1) является свойством автокорреляции, а уравнение. (2) — свойство взаимной корреляции. Большинство исследований было сосредоточено на случае, когда t a = t c = t , и в этом случае мы ссылаемся на ( n , w , t ) OOC. Опять же, можно изменить эти свойства в терминах (частичных) конструкций или упаковок. В этом случае OOC представляет собой набор из w подмножеств целых чисел (mod n ), так что для подмножеств c , b C ,

(3) c + i∩c + j≤tai ≠ j,

и

(4) c + i∩b + j≤tc.

Здесь c + i = { x + i (mod n ) ∣ x c }.

Код OOC эквивалентен циклическому расчету или упаковке. Код или упаковка называется циклической, если каждый циклический сдвиг кодового слова (или блока) является другим кодовым словом. Набор всех циклических сдвигов кодового слова называется орбитой . Представителя с этой орбиты часто называют базовым блоком. OOC ( n , w , t ) представляет собой набор базовых блоков для циклической (частичной) конструкции или упаковки S ( t + 1, w , n ) (при условии т < т ).И наоборот, при таком циклическом частичном S ( t + 1, w , n ) дизайн или упаковка, можно сформировать ( n , w , t ) OOC, взяв один репрезентативный блок или кодовое слово с каждой орбиты.

типов данных — как хранить однобайтовые целые числа в PostgreSQL?

Нет , в стандартном выпуске Postgres нет 1-байтового целого числа. Все встроенные числовые типы стандартных Postgres занимают 2 или более байта.

Расширение pguint

Но да , есть расширение pguint , поддерживаемое Питером Эйзентраутом, одним из разработчиков ядра Postgres. Не входит в стандартную поставку:

Помимо различных целочисленных типов без знака, он также предоставляет 1-байтовое целое число, которое вы ищете:

  int1 (8-битное целое число со знаком)
uint1 (8-битное целое число без знака)
uint2 (16-битное целое без знака)
uint4 (32-битное целое число без знака)
uint8 (64-битное целое число без знака)
  

Обязательно прочтите главу «Обсуждение» на связанном сайте, в которой объясняются возможные осложнения.Вам нужно проявлять осторожность с приведением типов и числовыми литералами при введении большего количества целочисленных типов …

Временное решение

Возможный простой обходной путь — закодировать 1-байтовые целочисленные значения как «char» (с двойными кавычками!), «Внутренний» упрощенный односимвольный тип, который фактически использует однобайтовых хранилища , байтовые значения однобайтового целого числа со знаком, верхняя половина которого представлена ​​в виде символов ASCII.

Вы можете кодировать значения в диапазоне от -128 до 127 .Демо:

  ВЫБРАТЬ i
     , i :: "char"
     , i :: "char" :: int
FROM generate_series (-128,127) i;
  

Есть несколько символов, не предназначенных для отображения. Итак, кодирует перед сохранением и декодирует перед отображением …

Помните: «char» — это «внутренний» тип, предназначенный для простого и дешевого перечисления. Официально не разработан для того, что мы здесь делаем, и не переносится на другие СУБД. Проект Postgres не дает никаких гарантий.Но поскольку "char" используется во всех системных каталогах, тип не изменится.

Изначально я предполагал беззнаковый 1-байтовый целочисленный диапазон (от 0 до 255) и использовал текст как ступеньку. Но это работает только для чисел от 1 до 127. Вместо этого используйте диапазон целых чисел со знаком (от -128 до 127) и приводите напрямую между «char» и целым числом .

Как получить максимальные и минимальные значения для целочисленных типов данных в зависимости от операционной системы? | Николас Тейшейра Ланза

Когда соучредитель вашей школы менее чем за 15 минут объясняет вам, как решить сложную задачу из учебной программы, в которой у вас мало опыта или знаний по этому вопросу, затем просит вас написать сообщение в блоге о том, как ОС машины находит максимальные и минимальные значения для целочисленного типа данных, вы с радостью соглашаетесь.

Если Жюльен это читает, то псевдокод, который на данный момент представляет собой царапину, является моей работой, а не работой другого ученика. Фотографировать код других студентов категорически не рекомендуется. Белая доска, с другой стороны, является краеугольным камнем обучения написанию программ.

Чтобы ответить на вопрос, во-первых, мы должны понять, какие типы данных существуют на используемом вами компьютерном языке (C для этого примера), подсчитать, сколько байтов находится в каждом типе данных, а затем выяснить, было ли приложение создано для 32-битный или 64-битный процессор.

На картинке выше есть уравнение, которое вы можете использовать в своей программе на языке C. Я расшифровал куриную царапину на картинке выше в уравнение, которое вы можете использовать ниже. Вы также можете скомпилировать и запустить этот файл, чтобы узнать размеры типов данных на 32-битном или 64-битном процессоре.

Беззнаковые типы данных:

int max = pow (2, количество битов, присвоенных типу данных) - 1;

Типы данных со знаком:

int min = (pow (2, количество битов, присвоенных типу данных) / 2) * -1;

int max = (pow (2, количество бит, присвоенных типу данных) / 2) - 1;

Какие типы данных существуют в языке C?

В GeeksforGeeks есть отличная статья обо всех типах данных на языке C.Не вдаваясь в архитектуру создания языка C, можно сказать, что каждому типу данных присвоены диапазоны значений. Читая картинку ниже, вы можете распознать закономерность.

Фото: GeeksforGeeks.

Слово int и char может выделяться вместе со словами со знаком и без знака . Слово int — это сокращение от целого числа, а char — сокращение от символа. При присвоении целочисленных значений типам данных в C существуют диапазоны значений, используемых в компьютерном языке C.Короткое int , имеющее два байта памяти, имеет минимальный диапазон значений -32 768 и максимальный диапазон значений 32 767 . unsigned short int , беззнаковое значение, не имеющее отрицательного знака (-), имеет минимальный диапазон 0 и максимальный диапазон 65 535 .

Фото: ASCIItable.com

Целочисленные типы данных просто выражаются как целые числа в определенном диапазоне с использованием приведенного выше уравнения. Типы данных char , с другой стороны, представляют собой символьные представления чисел, ссылающихся на таблицу ASCII.Взгляните на изображение таблицы ASCII выше. ASCII расшифровывается как Американский стандартный код для обмена информацией. Компьютеры производят вычисления с числами, поэтому код ASCII — это числовое представление символа, такого как символ «a», «x» или «!».

Когда вы объявляете переменную в своей программе, например

char first_initial = 'N';

вы назначаете целочисленное значение 78 таблицы ASCII вместо буквы 'N' переменной first_initial .Компьютер усваивает его как десятичное целое число 78 с основанием 10, которое затем выводится на терминал как символьное значение капитала N с помощью функции, подобной putchar (78) . Как разработчик программного обеспечения, вы должны принимать осознанные решения о том, какие типы данных вы хотите назначить в своем приложении, в зависимости от их использования памяти и типов проблем, которые вы пытаетесь решить.

Сколько битов в каждом типе данных?

Если вы снова обратитесь к таблице GeeksforGeeks, язык C имеет заранее определенное количество слотов памяти (байтов), назначенных каждому типу данных.В языке C 8 бит в 1 байте . Почему на языке C 8 бит были назначены на 1 байт?

Давайте вернемся к нашему присвоению заглавной буквы "N" нашей переменной. Когда мы назначаем 'N' типу данных char , мы назначаем 1 байт памяти переменной first_initial.

char — это 1 байт в C, потому что это определено в стандартах. Это потому, что двоичное представление типа данных char может уместиться в 1 байт.Во время первичной разработки C наиболее общедоступными стандартами символов были ASCII и EBCDIC , которые требовали 7- и 8-битного кодирования. 1 байт был достаточным для представления всего набора символов.

«Что такое двоичное представление типа данных char ?», — спросите вы.

Вы знаете знаменитую матричную картинку со всеми 0 и 1, которые окружают Нео, когда он участвует в своей эпической битве? Эти 0 и 1 отформатированы в двоичном представлении.

Двоичное представление — это система счисления с основанием 2, в которой для представления информации используются два взаимоисключающих состояния. Двоичное представление — это набор чисел, предназначенный для понимания компьютерами.

Что представляет собой это двоичное число в десятичной системе счисления?

0000 0110

Ответ — шесть. Как я пришел к такому выводу? Думайте о каждом числовом значении, представленном как бит. В приведенной выше числовой последовательности восемь числовых значений, 0 или 1, что означает восемь битов.0 , что равно 2 в нулевой степени.

Второе число будет 2 в 1-й степени, а затем 2 в 2-й степени, пока мы не окажемся в самом левом числе, которое будет равно 2 в 7-й степени. Ниже приведен простой дисплей, который показывает десятичное представление для каждой позиции двоичного числа.

128 64 32 16 8 4 2 1

0 0 0 0 0 1 1 0

Опять же, глядя на двоичное представление, мы можем использовать только значение 1 или 0, представляющее настоящее или отсутствует, потому что двоичный файл является взаимоисключающим.Если у нас есть 1 в значении, мы добавляем его к общей сумме, которую мы хотим представить в десятичной системе счисления. В нашем примере мы видим, что у нас есть 1 во 2-м десятичном представлении и еще 1 в 3-м десятичном представлении. Мы получаем 6, добавляя 2+ 4.

Еще одно полезное числовое представление для компьютеров — это шестнадцатеричное представление. Шестнадцатеричное представление — это еще один шаблон чисел, который представляет значение цвета для компьютера с использованием определенного шаблона нумерации, такого как # FF5733 , который имеет оранжевый цвет.

Итак, каким будет десятичное представление:

0110 0111 или 0011 1111 ?

Как получить максимальные и минимальные значения для целочисленных типов данных в зависимости от операционной системы на C? Используйте приведенное ниже уравнение.

Беззнаковые типы данных:

int max = pow (2, количество бит, присвоенных типам данных) - 1;

Типы данных со знаком:

int min = (pow (2, количество битов, назначенных типам данных) / 2) * -1;

int max = (pow (2, количество бит, назначенных типам данных) / 2) - 1;

Давайте использовать тип данных unsigned short int с максимальным диапазоном 65 535 .Мы можем найти это двумя способами. Самый простой способ — использовать приведенное выше уравнение. Чтобы найти максимальное значение для целочисленного типа данных без знака, мы возьмем 2 в степени 16 и вычтем на 1, что составит 65 535 . Мы получаем число 16, взяв количество байтов, присвоенных типу данных unsigned short int (2), и умножим его на количество бит, назначенных каждому байту (8), и получим 16.

Другой способ поиска out максимальное значение для типа данных unsigned short int в C использует двоичное представление.Мы знаем, что из приведенного выше примера тип данных unsigned short int состоит из 16 бит. Если мы поместим это в двоичное представление базы 2, это будет выглядеть примерно так.

1111 1111 1111 1111

В десятичной системе счисления это будет (1 + 2 + 4 + 8) + (16 + 32 + 64 + 128) + (256 + 512 + 1024 + 2048) + (4096 + 8192 + 16384 + 32768), что составляет 65 535 .

Способ представления короткого int со знаком отличается от способа представления короткого int без знака .Мы должны принять во внимание, что unsigned short int s — это только положительные числа. При использовании двоичного представления для положительных чисел каждый бит, равный 1, добавляется к представлению десятичного значения.

Отрицательные числа для 3-битных целых чисел со знаком. Фотография предоставлена ​​Исайей Беккер-Майер

Когда мы хотим использовать подписанный тип данных с использованием двоичного представления, если первый бит равен 0, число положительное, а если первый бит равен 1, число отрицательное, но мы считаем номера по-разному.

Мы считаем отрицательные числа иначе. Вместо того, чтобы начинать со всех 0 и считать 1, мы начинаем со всех 1 и считаем 0, начиная с -1. Взгляните на приведенный выше пример короткого int со знаком :

1000 0000 0000 0000

В приведенном выше примере -1 + -32,767 , что составляет -32,768 в десятичной системе счисления и является минимальным значением для подписанный короткий int типов данных. Опять же, отрицательные значения для подписанных типов данных начинаются со всех единиц и считаются нулями.Посмотрите на пример ниже.

1111 1111 1111 1100

В приведенном выше примере -4 в десятичной системе счисления. Теперь вы знаете, как найти максимальное и минимальное значения в двоичной системе счисления значений беззнаковых и подписанных типов данных.

32-битные и 64-битные процессоры

Существуют две принципиально разные архитектуры, используемые производителями компьютерных процессоров: 32-битная и 64-битная. Если вы хотите узнать больше о применимых различиях между ними, прочтите эту статью Джона Мартиндейла.

«Количество бит в процессоре относится к размеру обрабатываемых им типов данных и размеру его реестра. Проще говоря, 64-разрядный процессор более эффективен, чем 32-разрядный процессор, потому что он может обрабатывать больше данных одновременно. 64-разрядный процессор способен хранить больше вычислительных значений, включая адреса памяти, что означает, что он может получить доступ к более чем в четыре миллиарда раз большему объему физической памяти, чем 32-разрядный процессор. Это так же здорово, как кажется. 64-битные процессоры для 32-битных процессоров то же самое, что автомобиль для телеги.”

В зависимости от того, в какой ОС вы запускаете свою программу, язык C назначает разные байты разным типам данных. Скомпилируйте и запустите исполняемый файл из этого файла. Он напечатает что-то вроде этого:

Это показывает, что существуют разные назначения байтов для типов данных, которые выполняются на 32-битных процессорах и 64-битных процессорах.

Если у вас есть вопросы или комментарии, не стесняйтесь добавлять их ниже. Вы можете подписаться на меня в твиттере @ NTTL_LTTN. Спасибо за ваше время.

8, 16, 32, 64 и 128-битные целые числа - стенограмма видео и урока

Overflow / Underflow

Хорошо, давайте теперь поговорим о переполнении и недостаточном заполнении. Переполнение происходит, когда положительное значение превышает максимум, который может быть сохранен в типе данных, тогда как переполнение возникает, когда отрицательное значение меньше минимума, который может быть сохранен в типе данных. Выбор достаточно широкого типа данных, чтобы избежать вычислительного переполнения или потери значимости, имеет большое значение.Переполнение / недостаточное заполнение может привести к сбою программы и / или неправильному поведению, которое трудно диагностировать.

Рассмотрим пример, в котором переменная с именем NumberOfChairs объявлена ​​как 16-разрядный целочисленный тип данных. Наша компьютерная программа изначально устанавливает эту переменную в ноль, и каждый раз, когда в нашем инвентаре встречается другой стул, значение NumberOfChairs увеличивается на единицу.

Когда наш бизнес по производству стульев был небольшим, наш склад не мог вместить более 32 767 стульев.Однако со временем наш бизнес рос, как и наши складские возможности, и однажды неожиданно у нас на складе было 32 768 стульев. В зависимости от используемого языка программирования и / или процессора компьютера, прибавление 32 767 + 1 к 16-битному целому числу приведет либо к полному прерыванию программы, либо (что еще хуже) к нулевому значению, и программа продолжит работу.

В основном все идет хорошо, и без видимых причин наша программа обработки инвентаризации либо дает сбой, либо выдаёт неверные результаты без уведомления.Диагностика и устранение этого будут болезненными и дорогостоящими.

Практическое эмпирическое правило программирования состоит в том, чтобы выбрать тип данных, который может содержать значение, большее, чем вы думаете, реально может когда-либо возникнуть, а затем выбрать следующий тип данных большего размера. Печально известная ошибка тысячелетия (также известная как Y2K) стоила миллиарды долларов для исправления и была результатом того, что тысячи старых компьютерных программ выделяли только двузначное (то есть от 00 до 99 десятичное) значение для хранения, представляющее год, а не полное четыре цифры.

Вопросы эффективности

Итак, теперь, когда мы поговорили о переполнении и недостаточном количестве, давайте поговорим о соображениях эффективности. Большинство языков программирования требуют, чтобы перед использованием переменной был объявлен тип хранилища данных, связанный с целочисленным значением. После объявления тип данных (и, следовательно, размер его хранилища) не может быть изменен. Целочисленный размер данных по умолчанию (обычно, но не всегда, 32 бита) предполагается, если он не объявлен явно.

В тех редких случаях, когда арифметические операции должны выполняться с произвольно большими целыми значениями, могут быть вызваны специальные библиотеки «больших чисел».Эти библиотеки динамически расширяют и сжимают объем хранилища данных по мере необходимости для обработки значений в каждом вычислении. Поскольку этот метод значительно увеличивает время вычислений, их следует использовать только в случае крайней необходимости.

По мере развития электроники арифметической обработки, больше битов извлекается из памяти компьютера и обрабатывается в каждом машинном цикле, чем в прошлом. Ранние процессоры считывали / сохраняли только 8 бит за раз, поэтому выполнение операции с 32-битным типом данных заняло бы вдвое больше времени, чем с 16-битным типом данных.Однако многие процессоры сегодня извлекают / хранят 32 или 64 бита за раз, поэтому на самом деле может потребоваться такое же количество времени (в некоторых ситуациях, может быть, даже меньше) для выполнения операции с 32- или 64-битным значением, как и для операции с 32- или 64-битным значением. 16-битное значение.

В реальной практике программирования изменение скорости между типами данных обычно не является существенным фактором и в большинстве случаев может быть проигнорировано. Однако выбор типа данных, достаточно широкого, чтобы избежать вычислительного переполнения / потери значимости, чрезвычайно важен, потому что переполнение / недостаточное заполнение может привести к неверным результатам или катастрофическому сбою, в то время как медленная программа или излишне большая программа обычно продолжает работать правильно, хотя и не оптимально.

Итоги урока

Давайте рассмотрим! В большинстве случаев языки программирования требуют объявления типа данных, чтобы указать максимальный диапазон целочисленных значений, которые может представлять переменная. Помните, что количество из двоичных цифр (битов) определяет допустимый диапазон значений, которые могут быть сохранены. Важно выбрать подходящий размер, который не тратит лишнее пространство для хранения, но при этом может содержать все возможные значения, которые переменная может принимать как сейчас, так и в будущем.Предотвращение потери значимости или переполнения имеет решающее значение, поскольку это может привести к дорогостоящим и / или длительным программным сбоям.

В чем разница между типами данных byte и char в Java? Пример

Byte и char - это два числовых типа данных в Java, и оба могут представлять целые числа в диапазоне, но они сильно отличаются друг от друга. Основное различие между типом данных byte и char заключается в том, что байт используется для хранения необработанных двоичных данных, а другой используется для хранения символов или текстовых данных.Вы можете хранить символьные литералы в переменной типа char, например. char a = 'а'; Символьный литерал заключен в одинарные кавычки. Что касается диапазона, байтовая переменная может содержать любое значение от -128 до 127, но переменная типа char может содержать любое значение от 0 до 255.

Еще одно различие между byte и char в Java заключается в том, что размер байтовой переменной составляет 8 бит. в то время как размер переменной char составляет 16 бит. Еще одно различие между символом и байтом заключается в том, что байт может также представлять отрицательных значений , но char может представлять только положительных значений , поскольку его диапазон составляет от -128 до 127.

Другими словами, байт - это тип данных со знаком, где первый байт представляет знак числа, то есть 0 для положительного числа и 1 для отрицательного числа, но тип данных char беззнаковый. Давайте посмотрим еще на несколько различий между byte и char в Java.

Разница между типом данных byte и char в Java Одна из ключевых вещей, которые нужно знать о типах байтов и символов, - как преобразовать байт в char? Это область, в которой большинство программистов испытывают трудности, потому что вам нужна кодировка символов для сопоставления необработанных байтов с символом.Одинаковые байты будут отображаться на другой символ в другой кодировке символов, поэтому вы должны использовать ту же кодировку символов при преобразовании байта в char и наоборот.

В Java классы, выполняющие это преобразование, такие как FileReader или InputStreamReader, по умолчанию используют кодировку символов по умолчанию платформы, которая может быть правильной или неправильной. Вы также можете присоединиться к этим курсам программирования на Java, чтобы узнать больше о преобразовании байтов в символы в Java.

В любом случае, давайте посмотрим еще несколько моментов, чтобы лучше понять разницу между типом данных byte и char.

1) Первое и главное различие между byte и char заключается в том, что byte является типом данных со знаком, а char - типом данных без знака. В подписанном типе данных первый бит всегда представляет собой знак числа.

2) Из приведенного выше факта вы можете вывести еще одно различие между байтом и символом, состоящее в том, что первый может представлять отрицательные значения, но значения символов всегда положительны.

3) Еще одно различие между char и byte состоит в том, что char - это тип данных большего размера, чем byte. 16. - 1, что составляет 65535.


4) Вы можете инициализировать переменную типа char, используя символьный литерал, например. char ch = 'c', в этом случае значение ASCII символа 'c' будет сохранено в переменной char "ch".

5) Класс оболочки, соответствующий примитиву байта, - это java.lang.Byte, а класс оболочки, соответствующий примитиву char, - это класс java.lang.Character.

Вот и все, что касается разницы между типом данных byte и char в Java . Если вы знаете какое-либо другое отличие, которое, по вашему мнению, важно изучить, не стесняйтесь добавлять.Вы также можете проверить эти бесплатные курсы программирования на Java, чтобы узнать больше о различных типах данных и о том, когда их использовать в программах на Java, например. float, double, long, int, boolean и short.

побитовых операторов

побитовых операторов

Побитовые операторы

Биты и байты

Мы знаем, что память состоит из битов и байтов, и мы должны знать, что они уже есть.
  • бит - это наименьшая единица хранения в компьютере. Он хранит 0 или 1.
  • Байт состоит из 8 бит и является особенным, поскольку обычно наименьшая единица из непосредственно адресуемой памяти . Это означает - это наименьший элемент, из которого мы можем создать переменную. Адреса в памяти обычно применяются байты.
Самый маленький встроенный тип данных - char , который в большинстве систем сегодня 1 байт.
Итак ... что, если мы хотим получить доступ к отдельным битам? Это возможный? Да, но не напрямую.Мы должны использовать побитовое операторы действуют на битовом уровне.

Внимание : Побитовые операции могут быть машинно-зависимыми, чтобы степень, потому что они полагаются на способ, которым биты расположены в памяти. Некоторые машины могут не хранить все встроенные типы данных в одних и тех же форматах. Однако большинство машин обычно используют довольно стандартное представление для целочисленные типы, называемые форматом «дополнения до 2».

Побитовые операторы

Оператор Имя Артист Описание
и побитовое И двоичный Аналогично оператору &&, но побитно. побитовое исключающее ИЛИ двоичный Биты в результате устанавливаются в 1, если ровно один из соответствующих битов в двух операндах равно 1. Бит результата установлен в 0, если оба соответствуют биты в операндах одинаковы.
<< левый сдвиг двоичный Сдвигает биты первого операнда влево на количество биты, указанные во втором операнде. Справа заполните 0 битами.
>> правый сдвиг двоичный Сдвигает биты первого операнда вправо на количество биты, указанные во втором операнде.Левое заполнение зависит от машины. Обычно в зависимости от знака (заполните нулями положительные числа, единицы для негативов). y):
     х: 00011010 11101011
     г: 00101100 00110110
--------------------------
результат: 00110110 11011101 // это значение 14045

 
Вот побитовый сдвиг влево, выполняемый на x (x << 2):
      х: 00011010 11101011
---------------------------
сдвинуто: 01101011 10101100 // это значение 27564

 
Вот побитовый сдвиг вправо, выполняемый по y (y >> 4):
      г: 00101100 00110110
---------------------------
сдвинуто: 00000010 11000011 // это значение 707

 
А вот дополнение к x (~ x)
      х: 00011010 11101011
---------------------------
     ~ x: 11100101 00010100 // это значение -6892

 
Примеры кода

4.6 - Целые числа фиксированной ширины и size_t - Изучите C ++

В предыдущих уроках, посвященных целым числам, мы рассмотрели, что C ++ гарантирует только то, что целочисленные переменные будут иметь минимальный размер, но они могут быть больше, в зависимости от целевой системы.

Почему размер целочисленных переменных не фиксирован?

Если коротко, то это восходит к C, когда компьютеры были медленными, а производительность была главной проблемой. C решил намеренно оставить размер целого числа открытым, чтобы разработчики компилятора могли выбрать размер для int, который лучше всего работает на архитектуре целевого компьютера.

Разве это не отстой?

По современным меркам да. Программисту немного нелепо иметь дело с типами с неопределенными диапазонами.

Рассмотрим тип int. Минимальный размер int составляет 2 байта, но на современных архитектурах это часто 4 байта. Если вы предполагаете, что int составляет 4 байта, потому что это наиболее вероятно, ваша программа, вероятно, будет неправильно вести себя на архитектурах, где int на самом деле составляет 2 байта (поскольку вы, вероятно, будете хранить значения, требующие 4 байта в 2-байтовой переменной, что вызовет переполнение или неопределенное поведение).Если вы предполагаете, что int составляет всего 2 байта для обеспечения максимальной совместимости, то в системах, где int составляет 4 байта, вы тратите 2 байта на целое число и удваиваете использование памяти!

Целые числа фиксированной ширины

Для решения вышеуказанных проблем C99 определил набор целых чисел фиксированной ширины (в заголовке stdint.h), которые гарантированно будут одного размера в любой архитектуре.

Они определены следующим образом:

Имя Тип Диапазон Банкноты
std :: int8_t 1 байт со знаком -128 до 127 Во многих системах обрабатывается как знаковый символ.См. Примечание ниже.
std :: uint8_t 1 байт без знака 0 до 255 Во многих системах обрабатывается как символ без знака. См. Примечание ниже.
std :: int16_t 2 байта со знаком -32 768 до 32 767
std :: uint16_t 2 байта без знака 0 до 65 535
std :: int32_t 4 байта со знаком -2 147 483 648 до 2 147 483 647
std :: uint32_t 4 байта без знака 0 до 4 294 967 295
std :: int64_t 8 байт со знаком -9,223,372,036,854,775,808 до 9,223,372,036,854,775,807
std :: uint64_t 8 байт без знака 0 до 18 446 744 073 709 551 615

C ++ официально принял эти целые числа фиксированной ширины как часть C ++ 11.Доступ к ним можно получить, включив заголовок , где они определены внутри пространства имен std . Вот пример:

  #include  // для целых чисел фиксированной ширины
#include 

int main ()
{
    std :: int16_t я {5};
    std :: cout << i;
    возврат 0;
}  

Целые числа фиксированной ширины имеют два недостатка, которые обычно увеличиваются.

Во-первых, не гарантируется, что целые числа фиксированной ширины будут определены на всех архитектурах.Они существуют только в системах, где есть фундаментальные типы, соответствующие их ширине и следующие определенному двоичному представлению. Ваша программа не сможет скомпилироваться на любой такой архитектуре, которая не поддерживает целые числа фиксированной ширины, которые использует ваша программа. Однако, учитывая, что большинство современных архитектур стандартизировано около 8/16/32/64-битных переменных, это вряд ли будет проблемой, если ваша программа не должна быть переносимой на некоторые экзотические мэйнфреймы или встроенные архитектуры.

Во-вторых, если вы используете целое число фиксированной ширины, оно может быть медленнее, чем более широкий тип на некоторых архитектурах.Например, если вам нужно целое число, которое гарантированно будет 32-битным, вы можете решить использовать std :: int32_t , но ваш процессор может на самом деле быстрее обрабатывать 64-битные целые числа. Однако то, что ваш ЦП может обрабатывать данный тип быстрее, не означает, что ваша программа будет быстрее в целом - современные программы часто ограничены использованием памяти, а не ЦП, и больший объем памяти может замедлить вашу программу больше, чем более быстрый ЦП. обработка ускоряет его. Трудно узнать без точных измерений.

Быстрые и наименьшие целые числа

Чтобы помочь устранить указанные выше недостатки, C ++ также определяет два альтернативных набора целых чисел, которые гарантированно будут определены.

Быстрые типы (std :: int_fast # _t и std :: uint_fast # _t) предоставляют самый быстрый знаковый / беззнаковый целочисленный тип с шириной не менее # бит (где # = 8, 16, 32 или 64). Например, std :: int_fast32_t даст вам самый быстрый целочисленный тип со знаком, имеющий как минимум 32 бита.

Наименьшие типы (std :: int_least # _t и std :: uint_least # _t) предоставляют наименьший знаковый / беззнаковый целочисленный тип с шириной не менее # бит (где # = 8, 16, 32 или 64).Например, std :: uint_least32_t даст вам наименьший целочисленный тип без знака, длина которого не менее 32 бита.

Вот пример из авторской Visual Studio (32-разрядное консольное приложение):

  #include  // для целых чисел фиксированной ширины
#include 

int main ()
{
std :: cout << "минимум 8:" << sizeof (std :: int_least8_t) * 8 << "бит \ n";
std :: cout << "минимум 16:" << sizeof (std :: int_least16_t) * 8 << "бит \ n";
std :: cout << "минимум 32:" << sizeof (std :: int_least32_t) * 8 << "бит \ n";
std :: cout << '\ n';
std :: cout << "fast 8:" << sizeof (std :: int_fast8_t) * 8 << "бит \ n";
std :: cout << "fast 16:" << sizeof (std :: int_fast16_t) * 8 << "бит \ n";
std :: cout << "fast 32:" << sizeof (std :: int_fast32_t) * 8 << "бит \ n";

возврат 0;
}  

Это дало результат:

 минимум 8: 8 бит
минимум 16:16 бит
минимум 32:32 бит

быстро 8: 8 бит
быстро 16: 32 бит
быстро 32:32 бит
 

Вы можете видеть, что std :: int_least16_t - это 16 бит, тогда как std :: int_fast16_t на самом деле 32 бит.Это связано с тем, что на машине автора 32-битные целые числа обрабатываются быстрее, чем 16-битные.

Однако у этих быстрых и наименьших целых чисел есть свои недостатки: во-первых, не многие программисты действительно их используют, и незнание может привести к ошибкам. Во-вторых, быстрые типы могут привести к такой же потере памяти, которую мы видели с 4-байтовыми целыми числами. Наиболее серьезно, поскольку размер быстрых / наименьших целых чисел может варьироваться, вполне возможно, что ваша программа может демонстрировать различное поведение на архитектурах, где они разрешаются к разным размерам.Например:

  #include  // для целых чисел фиксированной ширины
#include 

int main ()
{
    std :: uint_fast16_t sometype {0};
    - некоторый тип; // намеренно переполнение, чтобы вызвать поведение зацикливания

    std :: cout << sometype;

    возврат 0;
}  

Этот код даст разные результаты в зависимости от того, является ли std :: uint_fast16_t 16, 32 или 64 бит.

Трудно понять, где ваша программа может работать не так, как ожидалось, пока вы не тщательно протестируете свою программу на таких архитектурах.И мы предполагаем, что немногие разработчики имеют доступ к широкому спектру различных архитектур для тестирования!

Предупреждение: std :: int8_t и std :: uint8_t могут вести себя как символы вместо целых чисел

Примечание. Подробнее о символах мы говорим в уроке (4.11 - Chars).

Из-за упущения в спецификации C ++ большинство компиляторов определяют и обрабатывают std :: int8_t и std :: uint8_t (и соответствующие быстрые и наименее фиксированные типы) идентично типам signed char и unsigned char соответственно.Следовательно, std :: cin и std :: cout могут работать иначе, чем вы ожидаете. Вот пример программы, показывающей это:

  #include 
#include 

int main ()
{
    std :: int8_t myint {65};
    std :: cout << myint;

    возврат 0;
}  

В большинстве систем эта программа будет печатать «A» (обрабатывая myint как символ). Однако в некоторых системах это может напечатать 65 , как и ожидалось.

Для простоты лучше вообще избегать std :: int8_t и std :: uint8_t (и связанных с ними быстрых и наименьших типов) (вместо этого используйте std :: int16_t или std :: uint16_t ).Однако, если вы используете std :: int8_t или std :: uint8_t , вы должны быть осторожны со всем, что могло бы интерпретировать std :: int8_t или std :: uint8_t как char вместо целого числа ( это включает std :: cout и std :: cin ).

Предупреждение

Избегайте 8-битных целочисленных типов фиксированной ширины. Если вы их используете, обратите внимание, что они часто рассматриваются как символы.

Интегральная передовая практика

Учитывая различные плюсы и минусы основных интегральных типов, целочисленных типов фиксированной ширины, быстрых / наименьших интегральных типов и проблем со знаком / без знака, единого мнения относительно передовых методов интеграции практически нет.

Наша позиция заключается в том, что лучше быть правильным, чем быстрым, лучше терпеть неудачу во время компиляции, чем во время выполнения - поэтому мы рекомендуем избегать быстрых / наименьших типов в пользу типов фиксированной ширины. Если позже вы обнаружите необходимость поддержки платформы, для которой типы фиксированной ширины не будут компилироваться, тогда вы сможете решить, как перенести свою программу (и тщательно протестировать) на этом этапе.

Лучшие практики

  • Предпочитайте int , когда размер целого числа не имеет значения (например,грамм. число всегда будет соответствовать 2-байтовому целому числу со знаком). Например, если вы просите пользователя ввести свой возраст или считаете от 1 до 10, не имеет значения, является ли int 16 или 32 бит (числа подходят в любом случае). Это покроет подавляющее большинство случаев, с которыми вы, вероятно, столкнетесь.
  • Предпочитайте std :: int # _t при сохранении количества, для которого требуется гарантированный диапазон.
  • Предпочитайте std :: uint # _t при выполнении битовых манипуляций или там, где требуется четко определенное циклическое поведение.

По возможности избегайте следующего:

  • Беззнаковые типы удерживаемых количеств
  • 8-битные целые типы фиксированной ширины
  • Быстрые типы с наименьшей фиксированной шириной
  • Любые специфичные для компилятора целые числа фиксированной ширины - например, Visual Studio определяет __int8, __int16 и т. Д.

Что такое std :: size_t?

Рассмотрим следующий код:

  #include 

int main ()
{
    std :: cout << sizeof (int) << '\ n';

    возврат 0;
}  

На машине автора это отпечатков:

 4
 

Довольно просто, правда? Мы можем сделать вывод, что оператор sizeof возвращает целочисленное значение, но какой целочисленный тип это возвращаемое значение? Int? Короткая? Ответ заключается в том, что sizeof (и многие функции, возвращающие значение размера или длины) возвращают значение типа std :: size_t .std :: size_t определяется как целочисленный тип без знака и обычно используется для представления размера или длины объектов.

Забавно, но мы можем использовать оператор sizeof (который возвращает значение типа std :: size_t ), чтобы запросить размер std :: size_t :

  #include  // std :: size_t
#include 

int main ()
{
std :: cout << sizeof (std :: size_t) << '\ n';

возврат 0;
}  

Скомпилировано как 32-битное (4 байтовое) консольное приложение в системе автора, выводится:

 4
 

Подобно целому числу, размер которого зависит от системы, std :: size_t также различается по размеру. std :: size_t гарантированно будет беззнаковым и имеет не менее 16 бит, но в большинстве систем будет эквивалентно ширине адреса приложения. То есть для 32-разрядных приложений std :: size_t обычно будет 32-разрядным целым числом без знака, а для 64-разрядного приложения size_t обычно будет 64-разрядным целым числом без знака.

Добавить комментарий

Ваш адрес email не будет опубликован. Обязательные поля помечены *